• Shuffle
    Toggle On
    Toggle Off
  • Alphabetize
    Toggle On
    Toggle Off
  • Front First
    Toggle On
    Toggle Off
  • Both Sides
    Toggle On
    Toggle Off
  • Read
    Toggle On
    Toggle Off
Reading...
Front

Card Range To Study

through

image

Play button

image

Play button

image

Progress

1/47

Click to flip

Use LEFT and RIGHT arrow keys to navigate between flashcards;

Use UP and DOWN arrow keys to flip the card;

H to show hint;

A reads text to speech;

47 Cards in this Set

  • Front
  • Back
A 38-year-old man is evaluated for persistent dyspepsia 2 months after a duodenal ulcer was detected and treated. He originally presented with new-onset epigastric pain, and esophagogastroduodenoscopy showed a duodenal ulcer; biopsy specimens showed the presence of Helicobacter pylori. The patient, who does not use NSAIDs and is penicillin-allergic, completed a 10-day course of therapy with omeprazole, metronidazole, and clarithromycin.

At this time, urea breath testing for H. pylori shows persistent infection.

In addition to a proton pump inhibitor, which of the following regimens is indicated for this patient?

A-Amoxicillin and levofloxacin
B-Bismuth subsalicylate, metronidazole, andtetracycline
C-Clarithromycin and amoxicillin
D-Clarithromycin and metronidazole
E-Trimethoprim–sulfamethoxazole and erythromycin
Correct Answer: B)

Educational Objective:Treat persistent Helicobacter pylori infection.
Key Point

* Bismuth-based quadruple therapy should be considered in a patient in whom initial proton pump inhibitor–based triple therapy has failed to eradicate Helicobacter pylori.

This patient has persistent Helicobacter pylori infection despite initial therapy with a proton pump inhibitor, clarithromycin, and metronidazole, an appropriate regimen for this patient with penicillin allergy. The most likely reason for failure of treatment in most patients is either noncompliance with therapy or antibiotic resistance; antibiotic resistance (probably to clarithromycin) is a likely cause of treatment failure in this patient. Therefore, an additional treatment regimen, one that does not contain clarithromycin, needs to be given. In the United States, bismuth-based quadruple therapy is indicated in a patient whose infection has failed to respond to proton pump inhibitor–based triple therapy. Levofloxacin-based triple therapy is also used in patients with persistent infection, but this regimen has not been validated in the United States.

Regimens containing amoxicillin would not be indicated in this patient with penicillin allergy. Therapy with the same regimen that failed initially to eradicate the organism because of likely antibiotic resistance would not be appropriate. H. pylori is naturally resistant to trimethoprim, and the regimen of trimethroprim–sulfamethoxazole and erythromycin is not an approved regimen for eradication of the organism.
A 19-year-old woman is evaluated for a 2-week history of nausea and new-onset jaundice. Six weeks ago she had an uncomplicated cystitis, which resolved after a 3-day course of therapy with trimethoprim-sulfamethoxazole.

On physical examination, the temperature is 37.3 °C (99.2 °F), the blood pressure is 120/85 mm Hg, the pulse rate is 88/min, and the respiration rate is 14/min; the BMI is 31. There is conjunctival icterus, jaundice, and right upper quadrant tenderness on deep palpation. Murphy sign is not elicited, and there is no asterixis or stigmata of chronic liver disease. Stool is negative for occult blood.

Laboratory studies:
Leukocyte count

7800/µL (7.8 × 109/L) with normal differential
Bilirubin (total)

12.0 mg/dL (205.2 µmol/L)
Bilirubin (direct)

5.6 mg/dL (95.6 µmol/L)
Aspartate aminotransferase

23 U/L
Alanine aminotransferase

35 U/L
Alkaline phosphatase

464 U/L
Antinuclear antibody titer

Negative
Anti–smooth muscle antibody

Negative
Antimitochondrial antibody

Negative

Ultrasonography of the right upper quadrant shows normal caliber of the hepatic ducts, a normal gallbladder without wall thickening, and no cholelithiasis.

Which of the following is the most appropriate management for this patient?

A-Cholecystectomy
B-Endoscopic retrograde cholangiopancreatography
C-Liver biopsy
D-Observation
Correct Answer: D)

Educational Objective:Evaluate drug-induced liver injury.
Key Point

* Liver test elevations in the setting of a recently started medication should raise the suspicion of a possible drug induced liver injury.

This patient has drug-induced liver injury secondary to trimethoprim-sulfamethoxazole therapy. Antibiotics are common causes of drug-induced liver injury, which can cause either an elevation in the aminotransferases or, as in this patient, a cholestatic form of liver injury. Some drugs have their own particular fingerprint of injury. For example, acetaminophen causes a predominant hepatocellular injury, whereas trimethoprim-sulfamethoxazole often produces a cholestatic form characterized by an alkaline phosphatase level more than twice normal. If a mixed pattern injury occurs in combination with an elevated alkaline phosphatase level, the patient is at increased risk of progressive liver injury. Phenytoin is an example of a drug that characteristically produces a mixed pattern. Drug-induced liver injury is often difficult to diagnose because there is no gold standard for diagnosis. Some drugs causing hepatic injury may be associated with the more familiar hypersensitivity syndrome characterized by fever, rash, and peripheral eosinophilia, but many drug reactions are characterized only by hepatic injury. Therefore, it is often a diagnosis of exclusion of other causes of liver injury in the presence of a potential offending agent taken within a recent period of time, usually weeks. Most patients can be monitored conservatively because discontinuation of the offending drug usually leads to eventual recovery, which can, however, take months. It is important to monitor for signs of progressive liver injury by monitoring prothrombin time and clinical status of the patient.

Cholecystectomy is incorrect because the patient has no radiographic evidence to suggest cholecystitis, her gallbladder is normal on imaging, she has no Murphy sign, gallstones, leukocytosis, or anything else to suggest cholecystitis. Endoscopic retrograde cholangiopancreatography (ERCP) is not correct because the patient has no clinical or radiographic suggestion of choledocholithiasis. Proceeding to ERCP in the absence of clinical suspicion is unlikely to be effective and places the patient at up to a 5% risk of post-ERCP pancreatitis. Liver biopsy is incorrect because there is a good clinical suspicion that the patient has drug-induced liver injury. If, however, her condition does not improve or if the diagnosis were less clear, then biopsy may be correct. Another time when liver biopsy might be appropriate is in the patient with hepatocellular injury who may have autoimmune hepatitis, which could respond to corticosteroid therapy.
A 35-year-old woman is evaluated for symptomatic ulcerative colitis. One year ago, she was diagnosed with pan-ulcerative colitis and responded well to initial and maintenance therapy with balsalazide. However, 2 months ago she developed urgent bloody diarrhea several times a day and lower abdominal cramping; prednisone, 40 mg/d, alleviated her acute symptoms, but her symptoms have returned with prednisone tapering. The patient is otherwise healthy, and her medications are balsalazide, 750 mg three times a day, prednisone, 15 mg/d, and calcium with vitamin D.

On physical examination, vital signs and other findings are normal. Laboratory studies reveal hemoglobin 11.4 g/dL (114 g/L) and plasma glucose 140 mg/dL (7.77 mmol/L). Stool analysis for Clostridium difficile toxin A and B is negative.

Which of the following is the most appropriate next step in the treatment of this patient?
AAdd olsalazine
BAdd budesonide
CAdd metronidazole
DIncrease prednisone dosage to 40 mg/d and add 6-mercaptopurine
Correct Answer: D)

Educational Objective:Treat ulcerative colitis.
Key Point

* Patients with ulcerative colitis who become corticosteroid-dependent should be started on therapy with an immunomodulator, such as azathioprine or 6-mercaptopurine with a steroid taper.

5-Aminosalicylates (5-ASA) are the first-line therapy for ulcerative colitis, and remission can often be induced and maintained with a 5-ASA only. When 5-ASA therapy is not effective initially or patients develop a flare while in remission on 5-ASAs, often a short course of corticosteroids is required to induce or re-induce remission. However, corticosteroids are not effective as maintenance therapy and have many potential side effects, including hyperglycemia, osteoporosis, hypertension, mood instability, acne, infection, and osteonecrosis. Although some patients may maintain remission with continued 5-ASA therapy after corticosteroid taper, other patients become corticosteroid-dependent or -resistant, as did this patient, and therapy with an immunomodulator such as 6-mercaptopurine or azathioprine should be started. These agents are nucleotide analogues that interfere with DNA synthesis and induce apoptosis. Therapy with these agents may be required for up to 3 months before providing clinical benefit, and therefore, they are generally started with corticosteroids, which are then tapered.

Because corticosteroids are not effective maintenance therapy, simply increasing the dose of prednisone without adding an immunomodulator would not be appropriate in this patient. The addition of another 5-ASA, such as olsalazine, will not provide any greater benefit. Antibiotic therapy has not been shown to be effective in the treatment of ulcerative colitis, and the patient’s stool was negative for Clostridium difficile. Budesonide is a nonsystemic corticosteroid that is useful in the induction of remission in patients with ulcerative colitis disease involving the terminal ileum and right colon, but would not be of added benefit in this patient with pan-ulcerative colitis.
A 27-year-old woman with an 8-year history of ulcerative colitis is evaluated during a follow-up examination. The initial colonoscopy after diagnosis showed pancolitis. She has been treated with mesalamine since diagnosis and has had episodes of bloody diarrhea two or three times a year but has otherwise done well. Her most recent colonoscopy 1 year ago when she had increased diarrhea and bleeding showed no progression of disease. Since then she has been clinically stable. The patient’s medical history includes nephrolithiasis, and her only medications are mesalamine, 2.4 g/d, and a multivitamin. There is no family history of inflammatory bowel disease or colorectal cancer.

On physical examination, vital signs are normal; BMI is 20.5. There is mild abdominal tenderness in the right lower quadrant without rebound or guarding. The rest of the physical examination is normal. Laboratory studies reveal a normal complete blood count, including leukocyte differential, and a serum C-reactive protein level of 0.8 mg/dL (8 mg/L).

Which of the following is the most appropriate management of this patient’s risk for colorectal cancer?
AAnnual capsule endoscopy
BAnnual flexible sigmoidoscopy
CColonoscopy now and annually thereafter
DIncreasing the dose of mesalamine to 3.6 g/d
Correct Answer: C)

Educational Objective:Manage colon cancer risk in a patient with inflammatory bowel disease.
Key Point

* Patients with inflammatory bowel disease should initiate screening for colorectal cancer after 8 years of duration of disease.

This patient has pancolitis of 8 years’ duration. The inflammation involves the ileum and proximal colon. The colon cancer risk in patients with ulcerative colitis or Crohn disease reaches a significant level after 8 years of inflammation; the annual cancer risk is estimated to be 1% to 2% per year after 8 years. The cancer risk is slightly delayed for patients with inflammation limited to the distal colon. The recommendation is to initiate a surveillance program with colonoscopy 8 years after onset of her disease, with follow-up colonoscopy every 1 to 2 years thereafter. Biopsies are performed in four-quadrant fashion throughout the entire colon.

The patient’s disease is reasonably well controlled on her current dose of mesalamine, and treatment with mesalamine does not in itself prevent colon cancer. There is no recommendation for standard screening for small-bowel carcinoma in the setting of ulcerative colitis or Crohn disease, and therefore, capsule endoscopy is not indicated. Flexible sigmoidoscopy would not reach the at-risk colonic mucosa in the proximal colon beyond the reach of the sigmoidoscope.
A 71-year-old man is evaluated for chronic epigastric discomfort, heartburn, and diarrhea of 4 years’ duration. His weight has been stable during this time period. The patient has no significant medical history and takes no medications.

On physical examination, there is mild epigastric tenderness but no rebound or guarding. Rectal examination reveals brown stool that is positive for occult blood. Laboratory studies reveal hemoglobin of 12.3 g/dL (123 g/L) with a mean corpuscular volume of 75 fL. Test for serum Helicobacter pylori antibody is negative. Esophagogastroduodenoscopy shows prominent gastric folds, mild linear erosions, and multiple ulcers in the stomach and the duodenum.

Which of the following is the most appropriate next step in the evaluation of this patient?
ACT scan of the abdomen and pelvis
BEndoscopic ultrasonography
CMeasurement of serum gastrin
DSomatostatin receptor scintigraphy
Correct Answer: C)

Educational Objective:Diagnose gastrinoma.
Key Point

* The first step in the evaluation of suspected gastrinoma is measurement of serum gastrin; serum gastrin concentration greater than 1000 pg/mL (1000 ng/L) is highly suggestive of gastrinoma.

Gastrinoma is a neuroendocrine tumor that secretes the hormone gastrin, which in turn results in hypersecretion of acid. Gastrinoma should be suspected when ulcers are present in a patient who has a negative test for Helicobacter pylori and who does not take NSAIDs. The initial test when gastrinoma is suspected is measurement of serum gastrin; a gastrin concentration of 1000 pg/mL (1000 ng/L) or greater is highly suggestive of a gastrinoma, but gastrinoma may be present with gastrin levels as low as 150 to 200 pg/mL (150 to 200 ng/L).

After the diagnosis of gastrinoma is made by detecting hypergastrinemia, various tests are used to localize the tumor and to evaluate for metastases. CT scan of the abdomen and pelvis is often the first test but may miss small pancreatic gastrinomas. Endoscopic ultrasonography is the most sensitive test for pancreatic endocrine tumors, including gastrinoma, but may miss tumors that are not localized within the pancreas. Somatostatin receptor scintigraphy is the most sensitive procedure for identifying the primary gastrinoma and detecting whether metastatic disease is present.
A 44-year-old woman is evaluated for a 6-month history of dyspepsia, regurgitation of sour fluid, and eructation. There is no associated fever, chills, weight loss, or vomiting. The condition failed to respond to a 6-week trial of omeprazole therapy. The patient’s medical history includes hypertension, type 2 diabetes mellitus, and obesity (BMI 36); her medications are lisinopril, metformin, and insulin glargine.

On examination, vital signs are normal; there is mild epigastric tenderness without rebound, and stool is negative for occult blood.

Which of the following is the most appropriate next diagnostic step in the evaluation of this patient?
AAmbulatory esophageal pH monitoring
BBarium esophagography
CCT scan of the chest
DEsophageal manometry
lCorrect Answer: A)

Educational Objective:Diagnose gastroesophageal reflux disease.
Key Point

* In patients with a high pretest probability of gastroesophageal reflux disease who fail to respond to a therapeutic trial of a proton pump inhibitor, ambulatory esophageal pH monitoring is most useful.

This patient’s clinical presentation is highly suggestive of gastroesophageal reflux disease (GERD). The typical symptoms of pyrosis, along with obesity, make the pretest probability of GERD high, although the lack of response to proton pump inhibitor therapy argues against the diagnosis. In patients with a high pretest probability of GERD who fail to respond to a therapeutic trial, ambulatory esophageal pH monitoring is most useful. Ambulatory pH monitoring employs a narrow-gauge cannula placed transnasally into the distal esophagus. Simultaneous pH measurement is made by several detectors. An endoscopically placed ambulatory monitor is becoming more commonly used. Abnormal esophageal acid exposure is determined by the percentage of time that esophageal pH is less than 4.0, esophageal acid exposure time, number of reflux episodes, mean duration and number of reflux episodes, and longest duration of reflux. The patient can record time of meals, episodes of reflux symptoms or pain, supine position, and other parameters to correlate symptoms to actual abnormal esophageal acid exposure.

Barium esophagography is a useful initial test in the evaluation of oropharyngeal reflux and to determine the presence of strictures but is less useful in determining significant episodes of gastroesophageal reflux or their relationship to symptoms. CT scan of the chest is performed in cases of dysphagia to investigate potential masses in the region of the gastric cardia. Esophageal manometry is useful in esophageal motility disorders such as diffuse esophageal spasm, achalasia, and hypotonic motility disorders. Newer techniques of esophageal function include esophageal impedance monitoring, which provides information on esophageal bolus transit as opposed to manometry’s ability to detect esophageal contractile activity.
A 71-year-old woman is evaluated for a 2-day history of progressive dyspnea on exertion. She has also had two episodes of black, tarry stool in the past week. She has not had fever, chills, cough, or abdominal pain or bright red rectal bleeding. The patient has a history of osteoarthritis for which she takes ibuprofen, 600 mg twice daily.

On physical examination, the temperature is 37.0 °C (98.6 °F), the blood pressure is 136/84 mm Hg, the pulse rate is 78/min, and the respiration rate is 12/min; the BMI is 24. Cardiac examination shows a grade 2/6 early systolic murmur at the base and regular rhythm with normal heart sounds. The lungs are clear, and there is no peripheral edema. Rectal examination reveals brown stool that is positive for occult blood. Laboratory studies reveal hemoglobin of 9.8 g/dL (98 g/L) with a mean corpuscular volume of 80 fL; serum biochemistry tests, including liver chemistry tests, and prothrombin time, activated partial thromboplastin time, and INR are normal. Chest radiography and echocardiography are normal; esophagogastroduodenoscopy, colonoscopy, and push enteroscopy are normal. Small-bowel capsule endoscopy shows a nonbleeding white ulcer in the mid-ileum.

Which of the following is the most appropriate next step in the management of this patient?
ADiscontinue ibuprofen therapy
BDouble-balloon enteroscopy
CEstrogen/progesterone therapy
DMesenteric angiography
EOctreotide therapy
Correct Answer: A)

Educational Objective:Manage a bleeding ulcer of the small bowel.
Key Point

* NSAID-induced injury to the bowel is a relatively common cause of small-bowel ulceration and may present with obscure gastrointestinal bleeding.

Ulceration of the ileum may occur as a result of infection, inflammation, or neoplasia. Medication-induced injury of the small bowel is a well-recognized complication of NSAID therapy; and discontinuing the medication results in ulcer healing and cessation of bleeding. Mild NSAID-induced gastrointestinal side effects include abdominal pain, nausea, constipation, diarrhea, and dyspepsia; major complications include ulcers, bleeding, perforation, and the attendant morbidity and morality. After NSAID therapy is discontinued, no further investigation is warranted if the patient’s bleeding ceases. Alternatively, ongoing blood loss or symptoms referable to the ulcer warrants further evaluation which may include directed biopsy and/or resection. The information from the capsule study regarding location of the bleeding site would direct the enteroscopy, which could be performed using the double-balloon technique to reach the site of injury.

Although estrogen/progesterone therapy and octreotide therapy may be useful for management of bleeding from enteric angiodysplasia, these therapies would not aid in ulcer healing. Mesenteric angiography would not identify an ulcer that is not actively bleeding and therefore would not be indicated in this patient.
A 41-year-old woman is evaluated for a 4-month history of intermittent mid-upper-abdomen pain, which does not radiate and is not affected by eating. She had gastroesophageal reflux when she was pregnant, but she says that the current symptoms are not like those of reflux or heartburn. She occasionally feels nauseated and mildly bloated, but she has not vomited, felt early satiety, or lost weight. She does not have difficulty swallowing or painful swallowing. Her bowel movements are normal. She has been pregnant twice and had two healthy children, both delivered by cesarean section. Her medical history also includes a cholecystectomy 5 years ago. Her only current medication is a multivitamin.

On physical examination, she is afebrile; the pulse rate is 65/min and the blood pressure is 110/65 mm Hg. There is no jaundice or scleral icterus; mild epigastric tenderness is present. Bowel sounds are normal; there are no abdominal bruits, palpable masses, or lymphadenopathy. Complete blood count and liver chemistry tests are normal.

Which of the following is the most appropriate next diagnostic test in the evaluation of this patient?
AAbdominal ultrasonography
BEsophagogastroduodenoscopy
CGastric scintigraphy
D Helicobacter pylori stool antigen
ESmall-bowel radiograph
Correct Answer: D)

Educational Objective:Evaluate dyspepsia.
Key Point

* In patients younger than 55 years who have new-onset dyspepsia without alarm symptoms, a “test and treat” approach for Helicobacter pylori is recommended.

This patient has new-onset uninvestigated dyspepsia. In a patient younger than 55 years without alarm features, the recommendation is to “test and treat” for Helicobacter pylori infection. The H. pylori stool antigen test detects active infection and therefore can be used in the initial diagnosis as well as for eradication testing.

An abdominal ultrasonography can be used in the evaluation of presumed biliary colic or chronic mesenteric ischemia (the latter if Doppler evaluation is additionally performed). The likelihood of biliary disease in this patient who is post-cholecystectomy and who has normal serum chemistry tests is low. Chronic mesenteric ischemia is also very unlikely because her pain does not increase with eating, and she has not lost weight, has no known risk factors for atherosclerosis, and has no abdominal bruits. An esophagogastroduodenoscopy is not needed given her age and lack of alarm features, but the procedure could be considered if her symptoms persist after H. pylori treatment. Gastric scintigraphy is used in the evaluation of gastroparesis and should only be performed after mechanical obstruction has been ruled out with upper endoscopy or barium radiography. The patient’s predominant symptom is pain and she has not vomited or had early satiety, and therefore, dyspepsia is more likely than gastroparesis. Although this patient has had several abdominal surgeries putting her at risk for a small-bowel obstruction from adhesions, the fact that her pain does not worsen with eating, she has not vomited, has maintained normal bowel movements, and has normal bowel sounds make obstruction unlikely; therefore, a small-bowel radiograph would be unnecessary at this point.
A 38-year-old woman is evaluated for elevated results of liver chemistry tests detected in an evaluation for new-onset fatigue, joint pains, and jaundice. The patient recently started a job in a hospital and received a hepatitis B vaccination. She has a history of hypothyroidism, and her only medications are levothyroxine and a multivitamin. She has never used illicit drugs and does not drink alcohol. Her mother has rheumatoid arthritis.

On physical examination, the patient is afebrile; the blood pressure is 130/75 mm Hg, the pulse rate is 80/min, and the respiration rate is 14/min. The BMI is 26. There is scleral icterus; the rest of the examination is normal.

Laboratory studies:
Leukocyte count

3400/µL (3.4 × 109/L) with a normal differential
Bilirubin (total)

6.0 mg/dL (102.6 µmol/L)
Bilirubin (direct)

3.6 mg/dL (61.6 µmol/L)
Aspartate aminotransferase

890 U/L
Alanine aminotransferase

765 U/L
Alkaline phosphatase

120 U/L
Antinuclear antibody

Titer 1:40
Anti–smooth muscle antibody

Titer 1:640
Antimitochondrial antibody

Negative

Viral serologic tests are negative.

Which of the following is the most likely diagnosis?
AAcute cholecystitis
BAutoimmune hepatitis
CDrug-induced liver injury
DPrimary biliary cirrhosis
EPrimary sclerosing cholangitis
Correct Answer: B)

Educational Objective:Diagnose autoimmune hepatitis.
Key Point

* Laboratory findings in patients with autoimmune hepatitis include elevated serum aminotransferase values, hypergammaglobulinemia, mild hyperbilirubinemia, elevated serum alkaline phosphatase values, and the presence of autoantibodies.

This patient has autoimmune hepatitis, a disorder that occurs most commonly in girls and young women. Like this patient with hypothyroidism, many affected patients have other autoimmune disorders and a family history of autoimmunity. These patients usually present with vague symptoms. Fatigue, which occurs in 85% of patients, is the most common presenting symptom, followed by jaundice (46%), anorexia (30%), myalgias (30%), and diarrhea. On physical examination, 78% of patients have an enlarged liver. Others have a normal examination despite the presence of advanced disease. The results of liver chemistry tests can be quite elevated, with aminotransferase concentrations into the thousands, but typically less than 500 IU at presentation, with elevated bilirubin, often with near-normal alkaline phosphatase. Autoimmune serologic tests, specifically antinuclear antibodies, anti–smooth muscle antibodies, and antibody to liver/kidney microsome type 1 (anti-LKM1), may be positive but are not detected in up to 25% of patients. Antibody levels are not prognostic of the disease course.

Primary biliary cirrhosis is a chronic progressive cholestatic liver disease of unknown cause. It is an autoimmune disorder that occurs predominantly in women (80% to 90% of cases) between 40 and 60 years of age. The diagnostic triad associated with primary biliary cirrhosis includes a cholestatic liver profile, positive antimitochondrial antibody titers, and compatible histologic findings on liver biopsy. Serum alkaline phosphatase level is usually elevated ten times or more above normal. The patient’s near normal alkaline phosphatase concentration and negative antimitochondrial antibody essentially rule out primary biliary cirrhosis. Although drug-induced liver injury can cause similar liver test abnormalities, the patient has not taken any new medications recently, making this diagnosis unlikely. She has no pain to suggest cholecystitis. Primary sclerosing cholangitis is a chronic cholestatic liver disease of unknown cause that is characterized by progressive bile duct destruction and may lead to secondary biliary cirrhosis. Laboratory findings include a cholestatic liver profile, with serum alkaline phosphatase levels three to five times greater than normal and mild hyperbilirubinemia. This patient’s alkaline phosphatase level is minimally elevated making primary sclerosing cholangitis unlikely.
A 32-year-old man is evaluated in the emergency department for a 5-day history of worsening crampy abdominal pain and eight to ten loose bowel movements a day. The patient has a 5-year history of ulcerative colitis treated with azathioprine and topical mesalamine; before this episode, he had one or two bowel movements of well-formed stool a day. The patient had sinusitis recently, which resolved with antibiotic therapy. He has otherwise been healthy and has not traveled recently, had contact with sick persons, or been noncompliant with medication.

On physical examination, the temperature is 38.3 °C (101 °F), the blood pressure is 130/76 mm Hg sitting and 105/60 mm Hg standing, the pulse rate is 90/min sitting and 120/min standing, and the respiration rate is 18/min. The abdomen is diffusely tender without rebound or guarding. Laboratory studies reveal hemoglobin 12.3 g/dL (123 g/L), leukocyte count of 28,000/µL (28 × 109/L) with 15% band forms, and platelet count of 234,000/µL (234 × 109/L). Intravenous fluids are started and stool studies are obtained.

Which of the following is the most appropriate next step in the management of this patient?
AIncrease dosage of azathioprine
BStart oral vancomycin
CStart oral mesalamine
DSmall-bowel radiographic series
Correct Answer: B)

Educational Objective:Manage Clostridium difficile colitis.
Key Point

* Infectious causes should be considered in exacerbations of diarrhea in patients with inflammatory bowel disease.

This patient likely has Clostridium difficile antibiotic-associated colitis complicating his underlying inflammatory bowel disease. C. difficile is an anaerobic gram-positive rod that produces two toxins, both capable of damaging the mucosa of the colon and causing pseudomembranous colitis. Infectious diarrhea associated with C. difficile has emerged as a major public health concern and can be seen in patients with underlying inflammatory bowel disease. Whenever a patient with inflammatory bowel disease presents with a new flair, stool studies, including C. difficile toxin assay, should be done. This patient’s recent history of antibiotic use greatly increases his risk of C. difficile infection. The fever, orthostasis, leukocytosis, and abdominal tenderness in the setting of chronic immunosuppression are all signs that he needs to be hospitalized for further investigations (for example, CT scan to rule out toxic megacolon) and to start empiric therapy. Optimal therapy is orally administered metronidazole or vancomycin and should be initiated promptly for severely ill patients.

It would be unwise to increase his immunosuppression either by adding prednisone or increasing the azathioprine in the setting of possible infection. There is no role for evaluation of the small bowel mucosa with a small-bowel series in order to diagnose small-bowel inflammation.
A 34-year-old woman is evaluated for continued severe mid-epigastric pain that radiates to the back, nausea, and vomiting 5 days after being hospitalized for acute alcohol-related pancreatitis. She has not been able eat or drink and has not had a bowel movement since being admitted.

On physical examination, the temperature is 38.2 °C (100.8 °F), the blood pressure is 132/84 mm Hg, the pulse rate is 101/min, and the respiration rate is 20/min. There is no scleral icterus or jaundice. The abdomen is distended and diffusely tender with hypoactive bowel sounds.

Laboratory studies:
Leukocyte count

15,400/µL (15.4 × 109/L)
Aspartate aminotransferase

189 U/L
Alanine aminotransferase

151 U/L
Bilirubin (total)

1.1 mg/dL (18.8 µmol/L)
Amylase

388 U/L
Lipase

924 U/L

CT scan of the abdomen shows a diffusely edematous pancreas with multiple peripancreatic fluid collections, and no evidence of pancreatic necrosis.

Which of the following is the most appropriate next step in the management of this patient?
AEnteral nutrition by nasojejunal feeding tube
BIntravenous imipenem
CPancreatic débridement
DParenteral nutrition
Correct Answer: A)

Educational Objective:Manage severe acute pancreatitis.
Key Point

* Enteral feeding is the preferred route of providing nutrition in patients with severe acute pancreatitis.

This patient has moderate to severe acute pancreatitis and after 5 days remains febrile, continues to be in pain, and cannot take in any oral nutrition. The patient will likely have an extended period before being able to take in oral nutrition. Two routes are available for providing nutrition in patients with severe acute pancreatitis: enteral nutrition and parenteral nutrition. Enteral nutrition is provided through a feeding tube ideally placed past the ligament of Treitz so as not to stimulate the pancreas. Parenteral nutrition is provided through a large peripheral or central intravenous line. Enteral nutrition is preferred over parenteral nutrition because of its lower complication rate, especially a lower infection rate. A meta-analysis of six studies with 263 participants compared enteral nutrition with total parenteral nutrition. Enteral nutrition was associated with a significantly lower incidence of infections, reduced surgical interventions to control complications of pancreatitis, and a reduced length of hospital stay. In another randomized, controlled trial, enteral nutrition showed a trend towards faster attenuation of inflammation, with fewer septic complications, and also was a dominant therapy in terms of cost-effectiveness.

Imipenem therapy is only helpful in acute pancreatitis when there is evidence of pancreatic necrosis. Pancreatic necrosis is diagnosed by a contrast-enhanced CT scan that shows nonenhancing pancreatic tissue. In patients with noninfected pancreatic necrosis, antibiotics may decrease the incidence of sepsis, systemic complications (for example, respiratory failure), and local complications (for example, infected pancreatic necrosis or pancreatic abscess). Randomized, prospective trials have shown no benefit from antibiotic use in acute pancreatitis of mild to moderate severity but may lead to development of nosocomial infections with resistant pathogens. Similarly pancreatic débridement is recommended only in patients with pancreatitis and infected pancreatic necrosis.

■Acute pancreatitis can be divided into two broad categories: edematous, interstitial or mild acute pancreatitis and necrotizing or severe acute pancreatitis. Treatment varies depending on the severity of the condition.
■Mild pancreatitis is treated for several days with supportive care including pain control, intravenous fluids, correction of electrolyte and metabolic abnormalities, and nothing by mouth. The majority of patients require no further therapy, and recover and eat within three to seven days.
■In severe pancreatitis, intensive care unit monitoring and support of pulmonary, renal, circulatory, and hepatobiliary function may minimize systemic sequelae.
■Abdominal pain is often the dominant symptom. Adequate pain control requires the use of intravenous opiates, such as meperidine and fentanyl, usually in the form of a patient controlled analgesia pump.
■In patients with mild pancreatitis, recovery generally occurs quickly, making it generally unnecessary to initiate supplemental nutrition. Soft diet can be started after resolution of pain.
■In patients with severe pancreatitis, we recommend attempting to provide early enteral nutrition in the first 72 hours through a nasojejunal tube placed endoscopically or radiologically (Grade 1B). If the target rate is not achieved within 48-72 hours and if severe acute pancreatitis is not resolved, supplemental parenteral nutrition should be provided.
■The occurrence of pancreatic infection is a leading cause of morbidity and mortality in acute necrotizing pancreatitis. We suggest prophylactic imipenem or meropenem in patients with necrosis that involves more than 30 percent of the pancreas (Grade 2C). However, not all guidelines recommend the routine use of antibiotics and it is reasonable to withhold antibiotics unless there is clinical or microbiologic evidence of infection.
■We suggest the following algorithm, based upon clinical and CT findings, to direct percutaneous aspiration, antibiotic therapy, and minimally invasive or open surgical debridement as needed (Grade 2B).
■In patients with gallstone pancreatitis, we recommend early ERCP and sphincterotomy for those who have a high suspicion of cholestasis and those with cholangitis (Grade 1B). Cholecystectomy should be performed after recovery in all patients with gallstone pancreatitis.
A 42-year-old woman is evaluated for a 20-year history of constipation. She has approximately one or two bowel movements a week consisting of lumpy or hard stool. She strains at defecation and has a sense of incomplete evacuation after a bowel movement. She does not have bloody stool, abdominal pain or discomfort, weight loss, or diarrhea. She is otherwise healthy, and her only medication is an occasional over-the-counter laxative or stool softener.

On physical examination, vital signs are normal. The anorectal tone is normal, and on rectal examination, the patient is able to expel the examiner’s finger when asked to mimic a bowel movement. Laboratory studies are normal. Radiopaque marker study shows delayed transit time through the right colon.

Which of the following is the most likely diagnosis?
AChronic intestinal pseudo-obstruction
BConstipation-predominant irritable bowel syndrome
CPelvic floor dysfunction (dyssynergic constipation)
DColonic inertia (slow-transit constipation)
Correct Answer: D)

Educational Objective:Diagnose slow-transit constipation.
Key Point

* In a radiopaque marker study, patients with slow-transit functional constipation have prolonged transit of markers through the proximal colon, whereas patients with dyssynergic functional constipation have normal transit time but impaired rectal expulsion of the markers.

Functional constipation is a common complaint. To make the diagnosis, there should not be any secondary causes such as an underlying metabolic or neurologic disease, mechanical obstruction, or drug effect. The disorder is characterized by the presence of two of the following Rome III criteria for functional constipation (for at least 3 months): straining during ≥25% of defecations; lumpy or hard stools ≥25% of defecations; sensation of incomplete evacuation for ≥25% of defecations; sensation of anorectal obstruction/blockage for ≥25% of defecations; manual maneuvers to facilitate ≥25% of defecations; and less than three defecations per week. Patients should not meet the Rome criteria for irritable bowel syndrome, which consist of recurrent abdominal pain or discomfort at least 3 days a month in past 3 months associated with two or more of the following: improvement with defecation; onset associated with change in frequency of stool; and onset associated with change in form (appearance) of stool. Patients with functional constipation should also not have alternating loose stools and constipation. There are two types of functional constipation: slow-transit (colonic inertia) and dyssynergic. This patient has slow-transit functional constipation. Affected patients have prolonged transit of radiopaque markers through the proximal colon, which is likely the result of an abnormality in the enteric nerve plexus.

In dyssynergic functional constipation or pelvic floor dysfunction (also known as outlet delay), transit time through the colon is normal but rectal expulsion of radiopaque markers is impaired, often resulting from the loss of the coordinated relaxation (or paradoxical contraction) of the puborectalis muscle and external anal sphincter. A radiopaque marker study therefore helps to differentiate these two types of functional constipation. Anorectal manometry can also help distinguish the two types, but a simpler diagnostic maneuver is having the patient mimic a bowel movement and attempt to expel the examiner’s inserted finger; if the patient cannot do so, dyssynergia rather than slow transit is present. This patient does not meet criteria for irritable bowel syndrome. There is no underlying disease or radiographic evidence in this patient to support the diagnosis of chronic pseudo-obstruction.
A 60-year-old man hospitalized for advanced cirrhosis complicated by ascites and encephalopathy is evaluated for massive hematemesis and hypotension. The patient’s medications are spironolactone, furosemide, and lactulose.

On physical examination, the temperature is 35.6 °C (96 °F), the blood pressure is 80/50 mm Hg, the pulse rate is 146/min, and the respiration rate is 20/min. The patient has just vomited red blood and has large-volume ascites; the stool is brown and positive for occult blood. Laboratory studies show hemoglobin of 9 g/dL (90 g/L), platelet count of 60,000/µL (60 × 109/L), and INR of 3.

In addition to rapid volume resuscitation, which of the following is the most appropriate management of this patient?
AArteriography
BEsophagogastroduodenoscopy
CIntravenous nadolol
DMesocaval shunt
ETransjugular intrahepatic portosystemic shunt
Correct Answer: B)

Educational Objective:Manage upper gastrointestinal bleeding.
Key Point

* Antibiotics, endoscopic variceal band ligation, and intravenous octreotide are the first-line therapies for acute esophageal variceal bleeding.

The first step in the management of acute variceal hemorrhage is the restoration of the intravascular volume using a large bore peripheral intravenous line or a central line. Packed erythrocytes are used as need to replace blood loss and clotting factors are replaced as needed. Platelet transfusions may be indicated if values fall below 50,000/µL (50 × 109/L). In addition, this patient should undergo urgent esophagogastroduodenoscopy and band ligation of esophageal varices. Clinical studies have shown that sclerotherapy was superior to balloon tamponade alone, vasopressin alone, and a combination of vasopressin and balloon tamponade in controlling active variceal hemorrhage, preventing early rebleeding, and improving survival in patients with esophageal and gastroesophageal varices. Band ligation has been shown to be as effective as sclerotherapy for preventing early rebleeding. Therapy should also be started with intravenous octreotide, which reduces portal venous blood inflow through inhibition of the release of vasodilatory hormones and is more effective for controlling bleeding than placebo; however, its ultimate effect on survival is unknown. Bacterial infections are present in a sizable percentage of hospitalized patients with variceal bleeding and are associated with a high mortality rate. Clinical trials have demonstrated that the use of prophylactic antibiotics in these patients results in a reduction in infectious complications and possibly mortality.

Arteriography is not first-line therapy in patients with a variceal bleed from venous portal hypertension. Arteriography is reserved for patients with a presumed arterial source of bleeding as can be seen in peptic ulcer disease or tumors anywhere along the gastrointestinal tract. In such cases, arteriography can be used to identify and embolize the specific vessel involved. This method is usually reserved for cases in which the patient is actively bleeding and either endoscopic therapy has failed to stop the bleeding or the presence of active bleeding interferes with identification of the bleeding site and the patient is unstable. Mesocaval shunt is a surgical shunt that decompresses the portal system by diverting portal venous flow into the inferior vena cava, thus bypassing the liver. This type of shunt is rarely used for acute variceal bleeding as it carries a high intraoperative risk in an acutely decompensated patient with underlying liver disease and coagulopathy. Intravenous nadolol is not appropriate because this patient is hypotensive and needs endoscopic intervention rather than medical therapy. Lastly, transjugular intrahepatic portosystemic shunting (TIPS) should not be used without first performing an upper endoscopy with esophageal band ligation. Many variceal bleeds can be controlled with endoscopic therapy alone, obviating the need for urgent TIPS which can have significant comorbidities including hepatic decompensation, bleeding complications, or resultant encephalopathy.
A 48-year-old man is evaluated 3 weeks after presenting with epigastric pain and early satiety with no weight loss. He underwent an esophagogastroduodenoscopy that showed thickened and nodular gastric folds in the body of the stomach with patchy erosions in the antrum. Biopsy specimens of the nodularity showed low-grade B-cell lymphoma; specimens of the antrum showed chronic active gastritis and no Helicobacter pylori. Endoscopic ultrasonography showed no invasion through the wall of the stomach; CT scan of the abdomen showed only gastric wall thickening. The patient’s medical history includes intermittent heartburn and occasional headache; his medications include as-needed antacids and an NSAID.

On physical examination, vital signs are normal. Examination of the abdomen reveals mild epigastric tenderness but no hepatosplenomegaly or lymphadenopathy. Complete blood count and serum chemistry tests are normal.

Which of the following is the most appropriate next step in the management of this patient?
AEvaluation for HIV infection
BInitiation of systemic chemotherapy
C Helicobacter pylori stool antigen test
DMeasurement of serum tissue transglutaminase antibody
ESurgical resection of the tumor
Correct Answer: C)

Educational Objective:Evaluate for Helicobacter pylori infection in a patient with mucosa-associated lymphoid tissue (MALT) lymphoma.
Key Point

* Patients with mucosa-associated lymphoid tissue (MALT) lymphoma should be tested for Helicobacter pylori infection.

This patient has a mucosa-associated lymphoid tissue (MALT) lymphoma, a low-grade B-cell lymphoma arising from the mucosal lymphoid tissue of the gastrointestinal tract. Half of MALT lymphomas are indolent, and most are located within the stomach. MALT lymphomas are thought to arise from clonal lymphocyte expansion in patients with chronic active gastritis secondary to Helicobacter pylori infection; eradication therapy for H. pylori infection alone results in lymphoma regression in up to 80% of affected patients; therefore, testing for H. pylori by stool antigen test is indicated in this patient.

Patients with HIV infection are at increased risk for Burkitt lymphoma, which can occur in the gastrointestinal tract; however, these tumors tend to be large and are usually located in the terminal ileum or rectum rather than the stomach and are not associated with chronic active gastritis. Patients with MALT lymphoma who have increased extent of disease as evidenced by ulceration, invasion through the gastric wall, or lymphadenopathy, which were not present in this case, are more likely to require standard chemotherapy for lymphoma. Although intestinal lymphoma is a malignant complication of celiac disease, this form of lymphoma is of T-cell origin (also known as enteropathy-associated T-cell lymphoma), and therefore, measurement of serum tissue transglutaminase antibody is not necessary. Surgical resection would not be required in a patient with early-stage MALToma who may fully respond to H. pylori eradication alone.
A 56-year-old woman is evaluated for a 3-year history of progressive dysphagia for solids and liquids; she has had a 6.8-kg (15-lb) weight loss during this time. The dysphagia was initially intermittent, but recently swallowing almost all food or drink causes a feeling of chest tightness and discomfort with increasingly frequent regurgitation of undigested food. The dysphagia is sometimes alleviated by standing upright. Her medical history is significant only for hypertension, and her medications include lisinopril and a multivitamin.

On physical examination, the patient appears uncomfortable and restless; she is thin but does not have thenar wasting. She is afebrile; the blood pressure is 142/92 mm Hg, the pulse rate is 96/min, and the respiration rate is 22/min. The BMI is 23.

Barium esophagography shows a dilated esophagus with an air/fluid level and tapered narrowing of the distal esophagus. Esophagogastroduodenoscopy shows a dilated esophagus with retained food and a tight lower esophageal sphincter, which allowed passage of the endoscope.

Which of the following is the most likely diagnosis?
AAchalasia
BDiffuse esophageal spasm
CPeptic stricture
DScleroderma esophagus
Correct Answer: A)

Educational Objective:Diagnose achalasia.
Key Point

* Achalasia is a primary motility disorder of the esophagus and requires manometric diagnosis.

The patient’s history is typical for achalasia, an uncommon but important primary motility disorder of the esophagus. The barium study and endoscopic appearance described are typical for achalasia, but the diagnosis is confirmed manometrically with esophageal motility studies. The manometric diagnosis of achalasia usually includes an elevated lower esophageal sphincter pressure, failure of the lower esophageal sphincter to relax with swallowing, and diminished or absent peristalsis of the esophageal body.

Diffuse esophageal spasm typically presents with noncardiac chest pain. The diagnosis of diffuse esophageal spasm is made manometrically by the finding of more than 20% of swallows having simultaneous contractions in the distal esophagus. Peptic stricture would present with dysphagia, but would typically show a longer, non-tapered stricture on barium esophagography. Furthermore, peptic strictures seldom present with megaesophagus, as seen in this patient. Scleroderma esophagus leads to loss of esophageal motility and often severe reflux or distal esophageal strictures, not a dilated esophagus.

Treatment of achalasia is usually pneumatic dilatation of the esophagus or surgical myomectomy, the latter of which can be done laparoscopically. Pneumatic dilatation, even in experienced hands, is associated with a 5% to 10% risk of esophageal perforation. Botulinum toxin injection can afford relief of achalasia in patients who are not considered candidates for endoscopic or surgical interventions.
A 45-year-old woman is evaluated for a 2-week history of right leg and flank pain and a slight limp. The patient was diagnosed with Crohn disease at age 20 years when she was evaluated for abdominal pain, nausea, and vomiting, and small-bowel radiographic series revealed a stenotic area in her terminal ileum with proximal dilatation and an enteroenteric fistula. She had an elective ileocolic resection with a primary anastomosis when her disease proved to be refractory to corticosteroid therapy. She has since been pain-free but has required intermittent courses of antibiotics and mesalamine for diarrheal flares of disease. She has had routine colonoscopic examinations with colonic biopsies showing scattered aphthous ulcerations and biopsy specimens revealing non-necrotizing granulomas in the colon and neo-terminal ileum. Her most recent colonoscopy was 5 years ago. She is otherwise healthy and takes no medication.

On physical examination, the temperature is 38.0 °C (101.0 °F), the blood pressure is 120/80 mm Hg, and the pulse rate 90/min; there is right-sided abdominal tenderness and restricted painful extension of the right hip; otherwise, the range of motion of the hip is normal and without pain. She has an abnormal gait, favoring her right leg.

Which of the following is the most likely diagnosis?
AAvascular necrosis of the hip
BCrohn disease-related arthritis
COsteoporosis-related fracture of the femoral neck
DPsoas muscle abscess
Correct Answer: D)

Educational Objective:Diagnose extraintestinal manifestations of Crohn disease.
Key Point

* The inflammation in Crohn disease is transmural and can extend into any adjacent structure, including muscle.

The prevalence of extraintestinal manifestations in patients with inflammatory bowel disease ranges from 21% to 40%. In most large studies, the prevalence is higher in Crohn disease than in ulcerative colitis. The right colon and transverse colon can abut the right psoas muscle, and the transmural inflammation of Crohn disease can lead to extension into the surrounding tissues and abscess formation. In this patient, who has a history of stricturing disease with fistula formation, it is likely that the fever, pain, and limp are complications of penetrating disease.

Peripheral and axial musculoskeletal syndromes occur in approximately 30% of patients with inflammatory bowel disease and are considered part of the seronegative spondyloarthropathies. The course of peripheral arthritis tends to parallel the activity of the bowel disease. Isolated hip involvement would be an unusual extra-colonic manifestation of Crohn disease and it would not explain this patient’s physical examination findings.

Patients with inflammatory bowel disease are at increased risk for developing osteoporosis, secondary to the disease process itself and the medications commonly used for treatment (such as corticosteroids). The fracture risk in such patients is 40% greater than that of the general population. All patients with inflammatory bowel disease older than 65 years with a history of corticosteroid use for more than 3 months should undergo bone mineral density scan. Furthermore, both short- and long-term corticosteroid therapy is associated with avascular necrosis of the femoral head. However, this patient has not taken corticosteroids for 25 years, and neither steroid-related avascular necrosis nor osteoporosis is likely and cannot account for her fever and flank pain.
A 57-year-old woman is evaluated for a 1-month history of increased abdominal girth. The patient has a 15-year history of alcohol abuse, drinking a bottle of wine a day. She also has type 2 diabetes mellitus, hypertension, and hyperlipidemia, and her medications are metformin, hydrochlorothiazide, propranolol, simvastatin, and aspirin.

On physical examination, the temperature is 37.1 °C (98.8 °F), the blood pressure is 90/50 mm Hg, the pulse rate is 99/min, and the respiration rate is 13/min; the BMI is 21. Examination reveals scleral icterus; bulging flanks; a small umbilical hernia; caput medusae; spider angiomata of the face, arms, and chest; and mild asterixis. There is no abdominal tenderness.

Laboratory studies:
Leukocyte count

5200/µL (5.2 × 109/L) with a normal differential
Platelet count

65,000/µL (65 × 109/L)
INR

2.2
Bilirubin (total)

3.4 mg/dL (58.1 µmol/L)
Bilirubin (direct)

1.8 mg/dL (30.8 µmol/L)
Aspartate aminotransferase

120 U/L
Alanine aminotransferase

65 U/L
Alkaline phosphatase

196 U/L
Albumin

2.7 g/dL (27 g/L)
Creatinine

2.7 mg/dL (206.0 µmol/L)
Urinalysis

Normal

Blood cultures are negative. Ultrasonography of the abdomen shows massive ascites, patent vessels, no ductal dilatation, and a shrunken liver with no masses.

Which of the following is the most appropriate management for this patient?
ADiagnostic paracentesis
BCefotaxime and albumin
CFurosemide and spironolactone
DTransjugular intrahepatic portosystemic shunt
Correct Answer: A)

Educational Objective:Manage new-onset ascites.
Key Point

* An ascitic fluid absolute polymorphonuclear cell count greater than 250/mL is indicative of spontaneous bacterial peritonitis.

This patient should have a diagnostic paracentesis to confirm the etiology of the ascites and to rule out spontaneous bacterial peritonitis (SBP). All patients who present with ascites should have a diagnostic paracentesis to categorize the ascites as secondary to portal hypertension or to other causes. This patient should have a small amount of fluid sampled for cell count with differential. An absolute polymorphonuclear cell count greater than 250/mL is indicative of SBP. Also total protein should be measured as well as albumin to calculate the serum albumin to ascites gradient (SAAG). An SAAG greater than 1.1 is consistent with portal hypertension; an SAAG less than 1.1 can occur in patients with malignant ascites. Patients do not always have symptoms suggestive of SBP such as fever, abdominal pain, or leukocytosis; therefore, their absence should not preclude a paracentesis to rule out SBP. In addition this patient has an elevated serum creatinine, which poses the possibility of hepatorenal syndrome for which SBP is a significant risk. Therefore, it is important to determine whether she has SBP.

Furosemide and spironolactone therapy is not indicated in the setting of an elevated serum creatinine and possible hepatorenal syndrome, because diuresis can precipitate worsening hemodynamic status or renal failure. Ceftriaxone and albumin are appropriate therapy for spontaneous bacterial peritonitis but are not indicated unless a diagnostic paracentesis can confirm the diagnosis. TIPS is not first-line therapy for new-onset ascites.
A 25-year-old man is evaluated for a 6-month history of eight loose, nonbloody stools a day; he also has abdominal pain and small-joint arthritis. The patient has a 5-year history of Crohn disease, initially treated with corticosteroids but then maintained on azathioprine therapy. Small-bowel radiographic series shows no stricturing disease but some active jejuno-ileitis. Azathioprine metabolites were recently measured and were found to be at therapeutic concentration.

Which of the following is the most appropriate additional therapy for this patient?
A5-Aminosalicylate
BAnti-tumor necrosis factor α-inhibitor
CCalcineurin inhibitor
DCorticosteroids
Correct Answer: B)

Educational Objective:Treat immunomodulator-refractory Crohn disease.
Key Point

* Patients whose Crohn disease becomes refractory to immunomodulator therapy should be given anti-tumor necrosis factor-α inhibitor therapy.

The patient’s Crohn disease had been well controlled on therapy with the immunomodulator azathioprine but is no longer responding; the small-joint arthritis may also be a complication of azathioprine. The next agent that should be used is the biologic agent infliximab an anti-tumor necrosis factor-α inhibitor. This potent biologic agent can bring the disease into remission quickly while sparing the effects of systemic corticosteroids. However, there is an increased risk of lymphoma and solid tumors associated with infliximab therapy. Recently it has been reported that combined infliximab and azathioprine is associated with an increased risk of hepatosplenic T-cell lymphoma in pediatric patients. Therefore, combination therapy is not preferred in patients who are receiving fixed-interval infliximab (every 8 weeks). Azathioprine dosage should be tapered and therapy with this agent eventually discontinued. Early use of infliximab has been shown to be more effective than conventional management with corticosteroids for induction of remission.

Corticosteroids are not used as maintenance therapy in Crohn disease primarily because of their numerous short- and long-term complications. The patient’s disease has now failed to respond to the steroid-sparing immunomodulator, and there is no role for corticosteroids. Calcineurin inhibitors have not been shown to be efficacious in Crohn disease. Mesalamines are not effective in immunomodulator-refractory Crohn disease.
A 53-year-old man is evaluated after a recent episode of substernal chest pain. He was evaluated in the emergency department for chest pain, and serial electrocardiograms and measurement of cardiac enzymes showed no evidence of myocardial ischemia. An outpatient stress test also showed no evidence of myocardial disease. The patient has a history of medically controlled hypertension, and his only medication is amlodipine.

On physical examination, the patient appears healthy and vital signs are normal. Barium esophagography shows a segmented or “corkscrew” esophagus. Esophageal manometry shows simultaneous contractions in the distal esophagus with 50% of swallows.

Which of the following is the most appropriate therapy for this patient?
AEsophageal bougienage
BLaparoscopic myotomy
COral anticholinergic therapy
DOral proton pump inhibitor therapy
EPneumatic dilatation
Correct Answer: D)

Educational Objective:Treat diffuse esophageal spasm.
Key Point

* Diffuse esophageal spasm is frequently related to gastroesophageal reflux; empiric therapy with acid suppression is the recommended first therapeutic intervention.

This patient has chest pain resulting from diffuse esophageal spasm. Although the diagnosis can be suggested by clinical presentation and the distinctive appearance of the esophagus on barium study, the disorder can only be diagnosed on manometry characterized by the finding of more than 20% of swallows having simultaneous contractions in the distal esophagus. Although the treatment for diffuse esophageal spasm is not clearly defined or the subject of general expert agreement, it is clear that many cases result from uncontrolled gastroesophageal reflux. Consequently, an empiric trial of acid suppression is usually recommended as the first therapeutic maneuver.

Oral anticholinergic agents have had limited success in this disorder, and their use is associated with frequent side effects. The more aggressive options of esophageal dilatation and surgical intervention have been proposed in case reports but have not been shown to be generally useful. Such approaches are reserved for patients with severe symptoms refractory to medical therapy.
A 38-year-old man is evaluated for a 3-month history of bloating and increased frequency of defecation. He has four to six bowel movements a day, including nocturnal bowel movements several times a week. The stool tends to be loose but there is no blood or melena. He has significant bloating but no abdominal pain. The patient underwent a Roux-en-Y gastric bypass procedure 2 years ago for morbid obesity; he lost 45.5 kg (100 lb) after the procedure. His weight stabilized 1 year ago, but he has lost 2.2 to 4.5 kg (5 to 10 lb) in the past few months. His medical history also includes hypertension, which resolved after the bariatric surgery, and his medications include a daily multivitamin with iron and daily oral calcium, vitamin D, and vitamin B12 supplements. The patient’s sister has Crohn disease, and he has twin 15-month-old daughters who attend a daycare center.

On physical examination, the blood pressure is 110/62 mm Hg, and the BMI is 29. He has redundant abdominal skin and healed abdominal incisions. The abdomen is not distended or tender; bowel sounds are normal. There is normal rectal tone and no palpable rectal abnormalities or perianal disease.

Laboratory studies:
Hemoglobin

11.7 g/dL (117 g/L)
Mean corpuscular volume

103 fL
Vitamin B12

185 pg/mL (136.5 pmol/L)
Folate

30 ng/mL (67.9 nmol/L)
C-reactive protein

0.6 mg/dL (6.0 mg/L)
Tissue transglutaminase antibodies
(IgA and IgG)

Negative

Stool culture and examination for fecal leukocytes and ova and parasites on three stool specimens are negative. Colonoscopy with terminal ileal examination is grossly normal. Esophagogastroduodenoscopy shows altered anatomy consistent with the bypass procedure; small-bowel biopsy specimens are negative. CT enterography is normal.

Which of the following is the most likely diagnosis?
ACeliac disease
BCrohn disease
CGiardiasis
DIrritable bowel syndrome
ESmall intestinal bacterial overgrowth
Correct Answer: E)

Educational Objective:Diagnose small intestinal bacterial overgrowth as a complication of bariatric surgery.
Key Point

* Small intestinal bacterial overgrowth is a late complication of bariatric surgery and should be considered in patients presenting with diarrhea, bloating, and features of malabsorption after such surgery.

Patients who undergo bariatric surgery are at risk for specific early and late complications of the procedure; small intestinal bacterial overgrowth is a late complication that can manifest with bloating, diarrhea, and features of malabsorption. Having segments of small bowel excluded from the usual stream of gastric acid, bile, and proteolytic enzymes, which all act to decrease excess bacterial growth in the small bowel, is a risk factor for bacterial overgrowth. The diarrhea in bacterial overgrowth can be from deconjugation of bile salts from the intestinal bacteria leading to fat malabsorption as well as from decreased disaccharidase levels leading to carbohydrate malabsorption. Other clues in this patient include the macrocytic anemia, with a low serum vitamin B12 level (bacteria bind and consume vitamin B12 and cleave it from intrinsic factor) and an elevated serum folate level (intestinal bacteria synthesize folate). Even though the patient is taking oral vitamin B12 replacement, his level is still low, suggesting inadequate replacement versus excessive loss. Culture of the small intestine or breath testing can be done to substantiate the diagnosis; alternatively, an empiric trial of antibiotic therapy could be considered.

Although celiac disease should be considered in any patient with diarrhea and features of malabsorption, the negative serologic studies and negative small-bowel biopsy rule out the diagnosis. Irritable bowel syndrome should not cause nocturnal stools, weight loss, and vitamin derangements. Crohn disease is possible given the patient’s symptoms and family history of inflammatory bowel disease, but the negative endoscopic, histologic, and radiographic findings make this disorder unlikely. The patient has young children in diapers who attend a daycare center, and, therefore, giardiasis is in the differential diagnosis; however, stool ova and parasite testing on three occasions has been negative, with the yield for repeated testing close to 90%.
A 55-year-old man is evaluated for a 4-month history of frequent and urgent defecation with loose and bloody stool, mild abdominal cramping, and fatigue. He has up to eight bowel movements a day and often wakes at night with symptoms; before this episode he had one bowel movement a day with well-formed stool. He does not have fever, nausea, or vomiting, but he has lost 3 kg (7 lb). He has mild joint pain in his knees and ankles that also began 4 months ago, which is worse in the morning and resolves somewhat during the day. The patient is a former cigarette smoker but quit smoking 2 years ago. His medical history includes hypertension, and his only medication is hydrochlorothiazide.

On physical examination, vital signs are normal. There is mild lower abdominal tenderness without rebound or guarding; there are no palpable abdominal masses. Examination of the rectum shows gross blood. Laboratory studies reveal hemoglobin 12.3 g/dL (123 g/L) with a mean corpuscular volume of 76 fL. Fecal leukocytes are present, but stool analysis is negative for infection. Colonoscopy shows continuous erythema, friability, and loss of vascular pattern from the rectum to the splenic flexure; the rest of the colon and terminal ileum is normal. Histology shows cryptitis, crypt abscesses, and crypt architecture distortion.

Which of the following is the most likely diagnosis?
ACrohn colitis
BInfectious colitis
CIschemic colitis
DMicroscopic colitis
EUlcerative colitis
Correct Answer: E)

Educational Objective:Diagnose ulcerative colitis.
Key Point

* Chronic histologic changes such as crypt architectural distortion are key distinguishing features of ulcerative colitis as compared to other causes of colitis, such as infection.

This patient has mild to moderate left-sided ulcerative colitis based on his clinical presentation and endoscopic and histologic findings. His ex-smoking status, microcytic anemia, and the presence of arthritis, which is the most common extraintestinal manifestation of inflammatory bowel disease, further support the diagnosis. While many colitides can have overlapping clinical, endoscopic, and even histologic features, there are important differences to consider. Microscopic colitis presents with nonbloody diarrhea, and colonoscopy shows normal mucosa macroscopically and histology shows either increased intraepithelial lymphocytes (lymphocytic colitis) or an increased submucosal collagen layer (collagenous colitis). Bleeding is less often a feature of Crohn colitis, and endoscopic inflammatory changes are patchy and generally spare the rectum; histologic features, however, may be indistinguishable from those of ulcerative colitis. Infectious colitis usually presents with more acute symptoms, and chronic changes such as crypt architecture distortion are absent. Ischemic colitis also generally has a more acute course and spares the rectum because of the dual blood supply to this region.
A 44-year-old man with a long history of alcohol abuse is evaluated on the sixth day of hospitalization for acute pancreatitis. On admission to the hospital, he was afebrile, the blood pressure was 150/88 mm Hg, the pulse rate was 90/min, and the respiration rate was 16/min. Abnormal findings were limited to the abdomen, which was flat and tender to palpation without peritoneal signs. Bowel sounds were normal. Plain abdominal and chest radiographs were normal. Abdominal ultrasonography revealed a diffusely enlarged, hypoechoic pancreas without evidence of gallstones or dilated common bile duct. He was treated with aggressive intravenous hydration and opioid analgesia. For the past 2 days, the patient has had repeated febrile episodes, persistent severe abdominal pain, and increasing shortness of breath.

On physical examination, the temperature is 38.6 °C (101.5 °F), the blood pressure is 98/60 mm Hg, the pulse rate is 112/min, and the respiration rate is 22/min; oxygen saturation is 92% with the patient breathing oxygen 3 L/min. Breath sounds are decreased at the base of both lungs. The abdomen is distended and diffusely tender with hypoactive bowel sounds. Laboratory studies reveal leukocyte count of 19,800/µL (19.8 × 109/L), creatinine 1.4 mg/dL (106.8 µmol/L), amylase 388 U/L, and lipase 842 U/L.

Which of the following is the most appropriate next step in the evaluation of this patient?
ACT scan of the abdomen with intravenous contrast
BEndoscopic retrograde cholangiopancreatography
CEndoscopic ultrasonography
DStool chymotrypsin
Correct Answer: A)

Educational Objective:Diagnose pancreatic necrosis.
Key Point

* CT scan of the abdomen with intravenous contrast is the most sensitive test to diagnose pancreatic necrosis.

Pancreatic necrosis should be suspected in a patient with severe acute pancreatitis whose condition is not improving or is worsening after 5 days or more of treatment. Pancreatic necrosis on CT scan can be identified as unenhanced areas of the pancreas. Neither endoscopic retrograde cholangiopancreatography nor endoscopic ultrasonography can detect the presence of pancreatic necrosis in the setting of acute pancreatitis. Stool chymotrypsin can be measured when chronic pancreatitis is suspected to help evaluate for decreased pancreatic function.

Pancreatic necrosis is the most important predictor of poor outcome in acute pancreatitis. Patients who develop pancreatic necrosis should be given antibiotic prophylaxis, usually with imipenem. The necrosis should be sampled for the presence of infection, and if infection is present, surgical débridement is recommended.
A 55-year-old man is hospitalized for a 2-week history of jaundice and altered mental status. The patient has a 10-year history of alcohol dependence and has failed several attempts to stop drinking. His family reports that he had been drinking heavily every day until about 3 weeks ago.

On physical examination, the patient is confused and lethargic; the temperature is 38.0 °C (100.0 °F), the blood pressure is 90/60 mm Hg, the pulse rate is 120/min, and the respiration rate is 30/min. The BMI is 24. Examination reveals scleral icterus. There is no guarding on palpation of the abdomen. The liver edge is tender and easily palpable 3 cm below the right costal margin. There is no ascites, edema, or evidence of bleeding.

Laboratory studies:
Leukocyte count

17,000/µL (17 × 109/L)
Platelet count

103,000/µL (103 × 109/L)
Prothrombin time

26.2 s
INR

4.0
Bilirubin (total)

37.0 mg/dL (632.7 µmol/L)
Bilirubin (direct)

17.0 mg/dL (290.7 µmol/L)
Aspartate aminotransferase

98 U/L
Alanine aminotransferase

50 U/L
Alkaline phosphatase

230 U/L
Albumin

2.0 g/dL (20 g/L)
Ammonia

110 µg/dL

Chest radiograph is normal. Ultrasonography shows an enlarged, fatty liver, with no nodules, ascites, pericholecystic fluid, or bile duct dilatation. Blood and urine cultures are negative.

In addition to enteral nutrition, which of the following is the most appropriate management for this patient?
ACeftriaxone
BMethylprednisolone
CFresh frozen plasma
DLiver transplantation
Correct Answer: B)

Educational Objective:Manage severe alcoholic hepatitis.
Key Point

* Patients with severe alcoholic hepatitis, as defined by a discriminant function score of 32 or more, benefit from corticosteroid therapy.

This patient has severe alcoholic hepatitis. Excessive alcohol intake may cause liver disease directly or may increase the risk of an unfavorable outcome in patients with pre-existing liver disease. This patient with chronic alcohol abuse has a history of recent heavy alcohol use, elevated serum aspartate aminotransferase (AST) and alanine aminotransferase values (usually greater than 300 U/L) and a serum aspartate aminotransferase concentration that is greater than the alanine aminotransferase concentration in roughly a 2 to 1 ratio, elevated alkaline phosphatase concentration, jaundice, coagulopathy, and encephalopathy. Moreover, other causes of acute and chronic liver disease have been excluded. The severity of the presentation and poor prognosis are underscored by the presence of the very high serum bilirubin concentration, coagulopathy, and encephalopathy.

The mortality risk in this setting is calculated by the discriminant function as follows: (4.6 × [prothrombin time − control prothrombin time]) + serum bilirubin. A discriminant function score of greater than 32 identifies patients with a 50% mortality rate within 30 days and has been used to identify patients who have a survival benefit from corticosteroid therapy. In addition, nutrition therapy has been shown to improve survival in severely malnourished hospitalized alcoholic patients.

The presentation of alcoholic hepatitis may resemble infection because of the fever and leukocytosis; however, this patient’s chest radiograph and blood and urine cultures were negative. In the absence of active bleeding, fresh frozen plasma is not indicated. Although many patients with alcoholic hepatitis recover with appropriate therapy, it would be inappropriate to offer liver transplantation to an alcoholic patient who is not involved in rehabilitation counseling. Some transplant programs require abstinence of at least 6 months. This not only confirms a commitment to abstaining from alcohol but also allows time for improvement of the alcoholic hepatitis to the point that transplantation may not be needed. Antibiotic therapy has no role in alcoholic hepatitis. Fresh frozen plasma is not a priority in a patient with alcoholic hepatitis and a coagulopathy unless active bleeding is present.
A 64-year-old woman is evaluated in the emergency department for her second episode of painless bloody stool. Four weeks ago, she was evaluated in the hospital for maroon stool; the hemoglobin at that time was 2 g/dL (20 g/L) lower than previous complete blood counts; esophagogastroduodenoscopy was normal; colonoscopy showed some old blood but no active bleeding. The patient was observed for 2 days and discharged with instructions for outpatient follow-up. Her current episode consisted of two maroon stools, one this morning and one 2 hours ago. Nasogastric lavage yields coffee grounds that clear with 1 L of saline. The patient had a colonoscopy 2 years ago at which time a single adenomatous polyp was detected and removed. She has no personal or family history of bleeding or gastrointestinal malignancy and is otherwise healthy; she has no upper gastrointestinal symptoms, does not use alcohol, and her only medication is a multivitamin.

On physical examination, the temperature is 37.1 °C (98.7 °F), the blood pressure is 98/62 mm Hg, the pulse rate is 94/min, and the respiration rate is 14/min; the BMI is 23.5. There is no scleral icterus; examinations of the heart and lungs are normal. The abdomen is soft with increased bowel sounds but no hepatosplenomegaly. Rectal examination reveals the presence of maroon and red blood but no palpable masses or hemorrhoids. Laboratory studies reveal hemoglobin of 9.4 g/dL (94 g/L) with a mean corpuscular volume of 86 fL; leukocyte count with differential and platelet count are normal. Prothrombin time, activated partial thromboplastin time, and INR are normal, as are liver chemistry tests. Serum blood urea nitrogen is 34 mg/dL (12.1 mmol/L), and creatinine is normal.

Which of the following is the most appropriate next step in the evaluation of this patient?
AColonoscopy
BDouble-balloon enteroscopy
CEsophagogastroduodenoscopy
DTechnetium red blood cell scan
EWireless capsule endoscopy
Correct Answer: C)

Educational Objective:Evaluate a patient with recurrent blood loss.
Key Point

* Repeat upper endoscopy in a patient with obscure upper gastrointestinal bleeding will identify a bleeding source in a significant proportion of patients.

This patient presents with obscure overt bleeding in that blood loss is clinically apparent because of the maroon stool, but the source is not identified despite endoscopic investigation. The most appropriate next test would be to repeat upper endoscopy. Studies have shown that between one third and two thirds of sources of obscure upper gastrointestinal bleeding are found within the reach of the upper endoscope on repeat endoscopy. The missed lesions are most often bleeding erosions associated with hiatus hernia, gastroduodenal angiodysplasia, Dieulafoy lesions, gastric antral vascular ectasias, and peptic ulcers. Moreover, endoscopy offers therapeutic options if a lesion is found in addition to establishing a diagnosis.

A colonoscopy is less likely to be helpful because the blood detected on nasogastric lavage suggests an upper gastrointestinal source. Wireless capsule endoscopy may be helpful if a repeat upper endoscopy is normal to assess the remainder of the small bowel; however, in contrast to endoscopy, capsule endoscopy does not offer therapeutic options. A radiolabeled red blood cell scan would not be the next test of choice; the results are nonspecific and would delay diagnosis and therapy that could be delivered more rapidly with endoscopy. In double-balloon enteroscopy, deep intubation of the small bowel is achieved by using a two-balloon technique to anchor the endoscope and assist in advancement. The procedure is lengthy and requires prolonged sedation, although it offers diagnostic and therapeutic capabilities. This test is reserved until a more proximal source of bleeding has been excluded and wireless capsule endoscopy has been performed. The capsule would direct whether the double-balloon enteroscope is to be introduced via the mouth (for jejunal abnormalities) or anus (ileal abnormalities).
A 68-year-old man with a history of alcoholism is evaluated in the emergency department for a 7-month history of diarrhea during which he has noted an increased volume of stool and decreased consistency. He has had intermittent abdominal pain but not severe enough to prevent him from eating or drinking. He is not taking any medications.

On physical examination, he is afebrile; the blood pressure is 108/72 mm Hg, the pulse rate is 80/min, and the respiration rate is 16/min. The abdomen is soft with mild periumbilical tenderness but no distention.

Laboratory studies:
Aspartate aminotransferase

155 U/L
Alanine aminotransferase

88 U/L
Alkaline phosphatase

96 U/L
Bilirubin (total)

1.1 mg/dL (18.8 µmol/L)
Amylase

65 U/L
Lipase

70 U/L

CT scan of the abdomen shows calcifications but no mass. There is fat in the stool.

Which of the following is the most appropriate management for this patient?
AFiber
BCholestyramine
CLoperamide
DPancreatic enzymes
Correct Answer: D)

Educational Objective:Manage diarrhea secondary to chronic pancreatitis.
Key Point

* The treatment of pancreatic insufficiency is enzyme replacement therapy.

The patient has chronic pancreatitis, which results from alcohol use in approximately 70% of adult cases, and most such patients have consumed more than 150 g/d of alcohol over 6 to 12 years. He also has concomitant alcoholic liver disease. Although the use of pancreatic enzyme supplementation for pain relief has had mixed results in patients with alcohol-related chronic pancreatitis, its use for control of the steatorrhea that results from pancreatic insufficiency is well established. This patient’s diarrhea will likely resolve by this intervention.

Fiber will bulk-up the stool but will not address the malabsorptive cause of diarrhea. Cholestyramine will bind bile salts, which is not the patient’s primary problem. Loperamide will slow down gut motility but he will continue to have fat malabsorption without pancreatic enzyme replacement.
A 74-year-old woman is evaluated for 3 years of progressive dysphagia, first for solid foods and now for both solid foods and liquids; she has had frequent episodes of regurgitation of undigested food and has lost 6.8 kg (15 lb) during the past 6 months. Her medical history includes stenting of the left anterior descending coronary artery 1 year ago after which she has had symptomatic residual distal stenosis. She had a cerebrovascular accident 2 years ago and still has mild residual right hemiparesis. Her medications include metoprolol, clopidogrel, enalapril, aspirin, and hydrochlorothiazide.

On physical examination, the patient is thin (BMI 20) and appears ill, although not in distress. Vital signs are normal. Chest radiograph shows a dilated esophagus with an air/fluid level and changes of chronic aspiration in the right lung base. Barium esophagography shows “bird beak” narrowing of the distal esophagus and mega-esophagus with retained fluid in the esophageal body. Esophageal manometry shows aperistalsis of the esophageal body and incomplete lower esophageal sphincter relaxation with swallowing. On esophagogastroduodenoscopy, the endoscope passes through the lower esophageal sphincter without resistance; there are no masses in the esophagus or the gastric cardia.

Which of the following is the most appropriate therapy for this patient?
AAnticholinergic therapy
BBotulinum toxin injection
CLaparoscopic myotomy
DPneumatic dilatation
Correct Answer: B)

Educational Objective:Treat achalasia in a patient who is not a candidate for endoscopic or surgical therapy.
Key Point

* Botulinum toxin injection can afford relief of achalasia in patients who because of age or comorbidities are not candidates for endoscopic or surgical intervention.

This patient’s history is typical for achalasia, an uncommon but important primary motility disorder of the esophagus. Although endoscopic evaluation is required to investigate the possibility of a mass lesion leading to partial esophageal obstruction, the diagnosis of achalasia is made manometrically using esophageal motility studies. The manometric diagnosis usually includes an elevated lower esophageal sphincter resting pressure, failure of the lower esophageal sphincter to relax with swallowing, and diminished or absent peristalsis of the esophageal body. Radiologic findings of note in achalasia include a “bird’s beak” abnormality of the distal esophagus, widening of the esophagus, and less commonly, megaesophagus with an air/fluid level. CT of the chest can be employed to differentiate achalasia from pseudoachalasia, the latter mimicking the true motility disorder but caused by a mass lesion at the distal esophagus or gastric cardia.

Treatment of achalasia is usually pneumatic dilatation of the esophagus or surgical myomectomy, the latter of which can be done laparoscopically. Pneumatic dilatation, even in experienced hands, is associated with a 5% to 10% risk of esophageal perforation. Botulinum toxin injection can afford relief of achalasia in patients like this one who because of age or comorbidities are not candidates for endoscopic or surgical intervention. Botulinum toxin inhibits the release of acetylcholine from nerve endings and has been used successfully for decades to treat certain spastic disorders of skeletal muscle such as blepharospasm and torticollis. Anticholinergic therapy is not indicated for achalasia.
A 56-year-old woman is evaluated for iron-deficiency anemia detected during her annual examination. An esophagogastroduodenoscopy showed diffuse linear antral erosions and small clean-based ulcers; gastric biopsy specimens were negative for Helicobacter pylori, and small-bowel biopsy specimens were normal. A subsequent colonoscopy was normal. The patient’s medical history includes long-standing rheumatoid arthritis and diarrhea-predominant irritable bowel syndrome, and her medications include methotrexate, hydroxychloroquine, nabumetone (an NSAID); daily calcium, vitamin D, and iron supplements; and loperamide. The patient has taken NSAIDs for many years; she once tried to switch to celecoxib, but it did not provide sufficient pain relief.

On physical examination, vital signs are normal; she has slight joint swelling of the metacarpophalangeal and proximal interphalangeal joints consistent with rheumatoid arthritis. Abdominal examination is normal. Laboratory studies reveal hemoglobin of 10.6 g/dL (106 g/L).

Which of the following is the most appropriate management for this patient?
AAdd an H2-receptor antagonist
BAdd misoprostol
CAdd a proton pump inhibitor
DAdd sucralfate
Correct Answer: C)

Educational Objective:Manage NSAID-induced gastric injury in a patient who requires long-term NSAID therapy.
Key Point

* Proton pump inhibitors are the drugs of choice in NSAID-induced gastric injury whether or not NSAID therapy is discontinued.

This patient has gastric erosions and ulcerations consistent with NSAID-induced gastropathy, which would account for her iron-deficiency anemia. In most patients with NSAID-induced gastric injury, discontinuation of the NSAID in addition to starting proton pump inhibitor therapy, would be reasonable, but in certain patients (like this one who requires NSAID therapy for her rheumatoid arthritis), the best option is to start proton pump inhibitor therapy, which has been found to heal peptic ulcerations whether or not NSAID therapy is ongoing. In such patients, reducing the dose of the NSAID versus switching to either a selective COX-2 inhibitor, such as celecoxib, or an NSAID with a safer side effect profile, such as nabumetone, etodolac, or meloxicam, should be considered. This patient did not receive sufficient pain relief with celecoxib and she is already taking nabumetone; therefore, simply adding a proton pump inhibitor is indicated.

Although high-dose H2-receptor antagonists such as famotidine are thought to be helpful in preventing ulcers, these agents have not been first-line therapy since the advent of proton pump inhibitors because they are not as effective. Misoprostol can reduce NSAID-related ulcers, but it has a dose-related side effect of diarrhea and would therefore not be indicated in this patient with diarrhea-predominant irritable bowel syndrome. Sucralfate has been used for ulcer prophylaxis, but there is no good evidence for its use in patients with long-term NSAID use and current ulcers.
A 65-year-old man is evaluated in the emergency department for fever and abdominal pain. The patient has a history of diverticulitis, and his latest flare 2 months ago was treated as an outpatient with antibiotics. While improved, he still has residual pain in the left lower quadrant. He has no other significant medical history and has not traveled recently.

On physical examination, the temperature is 38.8 °C (101.8 °F), the blood pressure is 120/85 mm Hg, the pulse rate is 100/min, and the respiration rate is 18/min; the BMI is 25. There is right upper quadrant abdominal pain with normal bowel sounds and no peritoneal signs, scleral icterus, or ascites.

Laboratory studies:
Hemoglobin

14.2 g/dL (142 g/L)
Leukocyte count

17,000/µL (17 × 109/L) with 15% band forms
Platelet count

320,000/µL (320 × 109/L)
INR

1.0
Bilirubin (total)

1.6 mg/dL (27.4 µmol/L)
Bilirubin (direct)

0.6 mg/dL (10.3 µmol/L)
Aspartate aminotransferase

97 U/L
Alanine aminotransferase

86 U/L
Alkaline phosphatase

220 U/L

CT scan of the abdomen and pelvis shows a mass with a fluid level consistent with a hepatic abscess. Antibiotic therapy is begun.

Which of the following is the most appropriate next step in the management of this patient?
AEndoscopic retrograde cholangiopancreatography
BLobectomy
CMRI with gadolinium
DPercutaneous drainage of the hepatic lesion
Correct Answer: D)

Educational Objective:Manage hepatic abscess.
Key Point

* Hepatic abscess is usually managed with percutaneous drainage of the lesion and intravenous antibiotics.

This patient requires percutaneous drainage of the hepatic lesion, which is likely an abscess. He has a history of recurrent diverticulitis, which does not appear to have been adequately treated in the past. He now has fever, right upper quadrant pain, and a large fluid-filled lesion in his liver. The clinical manifestations of pyogenic liver abscess usually include fever and right upper quadrant abdominal pain. Most pyogenic liver abscesses are due to biliary tract infection. Intra-abdominal infection may result in liver abscesses from seeding via the portal vein, whereas systemic bacteremia may cause abscesses from seeding via the hepatic artery. Abscesses resulting from an intra-abdominal infection are frequently due to anaerobic organisms, and many liver abscesses are polymicrobial. Percutaneous aspiration is recommended for diagnosis and subsequent treatment of a suspected pyogenic liver abscess. Antibiotics may be started even before aspiration and should not interfere with culture results. Parenteral antibiotic therapy to cover enteric gram-negative bacilli, anaerobes, and enterococci is recommended until culture results are available. Antibiotics specific for the causative organisms are then started and are continued for 4 to 6 weeks. Oral agents can be instituted after the abscess is drained and the patient is afebrile.

Lobectomy is not required in this patient because in most cases, a hepatic abscess can be managed with the combination of antibiotics and percutaneous drainage. Endoscopic retrograde cholangiopancreatography (ERCP) would not be appropriate because there is no evidence that this patient has cholangitis or obstructive jaundice. Although he has abnormal liver chemistry tests, these can be explained by the presence of the large abscess rather than biliary disease, especially because the CT scan does not show any evidence of ductal dilatation to suggest obstructive jaundice or cholangitis, which might require ERCP. MRI with gadolinium is not appropriate because the diagnosis can be made on the information provided, and no further confirmatory imaging is needed. The patient needs to be treated, and antibiotics and percutaneous drainage are required at this time.
A 55-year-old woman is evaluated in the hospital for a 2-day history of epigastric abdominal pain, nausea and vomiting, and anorexia. The patient has no significant medical history and takes no medications.

On physical examination, the temperature is 38.0 °C (100.5 °F), the blood pressure is 124/76 mm Hg, the pulse rate is 99/min, and the respiration rate is 16/min. There is scleral icterus and a slight yellowing of the skin. There is mid-epigastric and right upper quadrant tenderness. There is no palmar erythema, spider angiomata, or other evidence of chronic liver disease.

Laboratory studies:
Leukocyte count

14,900/µL (14.9 × 109/L)
Aspartate aminotransferase

656 U/L
Alanine aminotransferase

567 U/L
Bilirubin (total)

5.6 mg/dL (95.8 µmol/L)
Amylase

1284 U/L
Lipase

6742 U/L

Abdominal ultrasonography shows a biliary tree with a dilated common bile duct of 12 mm and cholelithiasis but no choledocholithiasis.

Which of the following is the most appropriate next step in the management of this patient?
ACT scan of the abdomen and pelvis with pancreatic protocol
BEndoscopic retrograde cholangiopancreatography
CHepatobiliary iminodiacetic acid (HIDA) scan
DMagnetic resonance cholangiopancreatography
Correct Answer: B)

Educational Objective:Manage gallstone pancreatitis.
Key Point

* In patients with gallstone pancreatitis and evidence of biliary obstruction, endoscopic retrograde cholangiopancreatography and stone removal reduces morbidity and mortality by reducing the risk of biliary sepsis.

This patient has a classic presentation of acute pancreatitis with the acute onset of epigastric abdominal pain, nausea, and vomiting associated with markedly elevated pancreatic enzymes. The presence of stones in the gallbladder, a dilated bile duct, and elevated aminotransferase levels highly suggest gallstones as the cause of pancreatitis. The scleral icterus, jaundice, and elevated bilirubin level suggest continuing bile duct obstruction. Abdominal ultrasonography has a sensitivity of only 50% to 75% for choledocholithiasis, and a common duct stone should be suspected in the correct clinical situation even when ultrasonography does not show a stone.

Endoscopic retrograde cholangiopancreatography (ERCP) with sphincterotomy and stone removal is the most appropriate procedure in patients with acute gallstone pancreatitis and with imaging and biochemical evidence of biliary obstruction from a common duct stone. The procedure can document the diagnosis of choledocholithiasis and remove the gallstones, which lessens the morbidity and mortality due to biliary sepsis.

CT scan will likely show evidence of acute pancreatitis and magnetic resonance cholangiopancreatography (MRCP) will show pancreatitis and the presence of a common duct stone, with sensitivities for CT being approximately 75% for stones and MRCP having sensitivities ranging from 80% to 100% as compared with ERCP for the diagnosis of choledocholithiasis. Biliary scintigraphy may show obstruction of the cystic or common bile duct but will not determine the cause. However, CT, biliary scintigraphy, and MRCP will not be therapeutic for bile duct stones.
A 74-year-old man is evaluated in the hospital for severe diffuse abdominal pain. He was hospitalized 5 days ago for chest pain and was found to be in rapid atrial fibrillation and a myocardial infarction was diagnosed. He underwent cardiac catheterization and double stent placement after which he has had intermittent hypotension and has remained in atrial fibrillation with a controlled ventricular rate. At the bedside the patient is sweating, nauseated, and holding his abdomen. He cannot respond to questions. His wife says that he has never had any gastrointestinal problems, but that he has not had a bowel movement since he entered the hospital. The patient has chronic atrial fibrillation but he discontinued his anticoagulation therapy 6 months ago; he also has hyperlipidemia. His medications are heparin, metoprolol, simvastatin, clopidogrel, and aspirin.

On physical examination, the temperature is 38.0 °C (99.5 °F), the blood pressure is 102/60 mm Hg, the pulse rate is 94/min, and the respiration rate is 25/min. There is mild, diffuse abdominal tenderness to palpation without rebound or guarding; there are no palpable abdominal masses. Laboratory studies reveal only a leukocyte count of 13,000/µL (13 × 109/L). Radiograph of abdomen shows no evidence of perforation or obstruction.

Which of the following would be the most appropriate management for this patient?
ACT arteriography
BColonoscopy
CIntravenous famotidine
DLactulose
Correct Answer: A)

Educational Objective:Manage acute mesenteric ischemia.
Key Point

* CT angiography is the most sensitive diagnostic test for acute mesenteric ischemia.

This patient most likely has acute mesenteric ischemia, a potentially fatal condition that typically affects elderly patients. The disorder may result from embolic or thrombotic occlusions of the splanchnic vessels or may be nonocclusive. Superior mesenteric artery embolism is the most common cause, accounting for approximately 50% of cases and usually developing from ventricular or left atrial thrombi in patients with atrial fibrillation. This patient’s age, recent myocardial infarction, atrial fibrillation, history of discontinuing anticoagulation, and hypotension are all significant risk factors for this condition. His low-grade fever, hypotension, sudden onset of severe abdominal pain out-of-proportion to physical examination findings, as well as his elevated leukocyte count all support the diagnosis. CT arteriography is a sensitive test for acute mesenteric ischemia; the procedure can evaluate the mesenteric vessels and also assess for signs of bowel ischemia and rule out other potential causes of acute onset abdominal pain, such as perforation or obstruction. Surgical consultation should be done simultaneously because if the results of CT arteriography support the diagnosis of acute mesenteric ischemia, the patient may need emergent surgery. Traditional angiography is the gold standard for diagnosis of acute mesenteric ischemia and can also offer therapeutic options such as the use of vasodilators as well as balloon dilation and stent placement. Surgery is required for patients with peritoneal signs and/or evidence of bowel necrosis on imaging. Surgical interventions include peritoneal lavage, resection of necrotic and perforated bowel, thrombolectomy, patch angioplasty, endarterectomy, and bypass procedures. Successful thrombolytic therapy in stable patients without peritoneal signs has been reported.

Colonoscopy would not be appropriate for this patient, because it does not evaluate the mesenteric vessels or the small intestine, only the mucosa of the colon. Intravenous famotidine therapy for possible dyspepsia would also not be appropriate in this patient with a potentially life-threatening condition. Likewise, although the patient may be somewhat constipated, giving him lactulose only is not correct.
A 35-year-old woman with a history of chronic hepatitis C virus infection is evaluated 4 weeks after starting therapy with pegylated interferon and ribavirin. The patient also has a history of hypothyroidism, and her medications include interferon, ribavirin, and levothyroxine. The patient’s mother has rheumatoid arthritis and hypothyroidism.

On physical examination, the patient is afebrile; the blood pressure is 100/78 mm Hg, the pulse rate is 65/min, and the respiration rate is 12/min. The BMI is 26. There is scleral icterus; there is no edema, ascites, or spider angioma.

Laboratory studies:

1 Month Ago

Current
Bilirubin (total, mg/dL)

1.2 (20.5 µmol/L)

4.0 (68.4 µmol/L)
Bilirubin (direct, mg/dL)

0.3 (5.1 µmol/L)

2.3 (39.3 µmol/L)
Aspartate aminotransferase (U/L)

37

770
Alanine aminotransferase (U/L)

39

890
Alkaline phosphatase (U/L)

90

167
Albumin (g/dL)

4.0 (40 g/L)

3.6 (36 g/L)
Antinuclear antibody

Negative

Titer 1:80
Anti–smooth muscle antibody

Negative

Titer 1:640
Antimito-chondrial antibody

Negative

Negative
Hepatitis C virus RNA

580,000 copies/mL

540,000 copies/mL

Ultrasonography of the liver is normal.

Which of the following is the most appropriate next step in the management of this patient?
AContinue interferon and ribavirin at current dosages
BContinue interferon and hold ribavirin
CDiscontinue interferon and ribavirin
DDiscontinue interferon and ribavirin for 1 week
Correct Answer: C)

Educational Objective:Manage interferon-induced autoimmune hepatitis.
Key Point

* Interferon therapy can precipitate autoimmune hepatitis, and therapy should be discontinued in affected patients.

This patient is experiencing a flare of autoimmune hepatitis precipitated by interferon therapy. Such a flare can occur in patients with quiescent autoimmune hepatitis or as new-onset disease induced by interferon therapy. Other autoimmune diseases can develop or be exacerbated during interferon therapy, including rheumatoid arthritis, sarcoidosis, dermatitis, and type 1 diabetes mellitus. Accordingly, interferon therapy should be used cautiously in patients with autoimmune disease and is contraindicated in patients with pre-existing autoimmune hepatitis. This patient has worsening liver disease as well as established hypothyroidism; therefore, the interferon and ribavirin therapy should be discontinued and the patient monitored closely. If the autoimmune hepatitis does not stabilize, treatment for this disorder will have to be considered. This patient should not be rechallenged with hepatitis C therapy because it is likely to cause worsening liver disease.

Decreasing the dosage of the interferon and ribavirin is not an option, because the autoimmune response is not dose-related. Although some patients receiving therapy for hepatitis C experience a flare of hepatitis before clearing the virus, this patient has persistent viral replication, and given the newly positive antinuclear antibody and anti–smooth muscle antibody test results suggestive of autoimmune hepatitis and personal and family history of autoimmune disorders, therapy should be discontinued to avoid worsening hepatic decompensation.
A 30-year-old woman is evaluated for a 9-month history of cramping midepigastric discomfort that is relieved by defecation; the discomfort is sometimes accompanied by bloating. The stool is often watery. She has not had fever, chills, or weight loss. The patient is otherwise healthy and takes no medications; there is no family history of gastrointestinal disease.

On physical examination, the patient is afebrile; the blood pressure is 105/70 mm Hg, the pulse rate is 72/min, the respiration rate is 14/min, and the BMI is 23. The abdomen is soft and not tender or distended; the stool is brown and negative for occult blood. Complete blood count and serum biochemistry studies, including liver studies, vitamin B12, vitamin D, and thyroid-stimulating hormone, are normal.

Which of the following is the most appropriate management for this patient?
AColonoscopy
BCT enteroscopy
CGluten-free diet
DSymptomatic management
Correct Answer: D)

Educational Objective:Manage diarrhea-predominant irritable bowel syndrome.
Key Point

* Irritable bowel syndrome is a diagnosis of exclusion but in the absence of alarm symptoms, invasive work up is not necessary

This patient presents with symptoms that meet the Rome III criteria for irritable bowel syndrome (IBS). The Rome criteria were developed to establish consensus guidelines for diagnosis of functional bowel disorders. Criteria for IBS are symptoms of recurrent abdominal pain or discomfort and a marked change in bowel habit for at least 6 months, with symptoms experienced on at least 3 days a month for at least 3 months. Two or more of the following must also apply: (1) pain is relieved by a bowel movement; (2) onset of pain is related to a change in frequency of stool; and/or (3) onset of pain is related to a change in the appearance of stool. IBS is the most common gastrointestinal condition diagnosed in the United States. It is characterized by chronic abdominal pain and altered bowel habits in the absence of any organic cause.

In this otherwise healthy young woman, reassurance that she has a chronic but not a life-threatening disease with recommendation of a high-fiber diet should be the initial therapy. CT enteroscopy or colonoscopy would be premature at this point given the absence of alarm symptoms: fever, weight loss, blood in stool, abnormal physical examination, family history of inflammatory bowel disease or colon cancer, or pain or diarrhea that awakens/interferes with sleep.

This patient does not have evidence of malabsorption, anemia, or weight loss to suggest a diagnosis of celiac disease; therefore, an empiric gluten-free diet would be inappropriate.
A 32-year-old man is evaluated in the emergency department for chest discomfort of 2 hours’ duration and that occurred after he ate a large meal. He has difficulty swallowing and feels as though something is stuck in his chest; he is barely able to swallow his saliva and frequently spits into a cup. He has had two similar but less severe episodes in the past 6 months. The patient’s only significant medical history is a 20-year history of seasonal allergic rhinitis treated with nasal corticosteroids and oral antihistamines.

On physical examination, he is well developed and well nourished but uncomfortable. Esophagogastroduodenoscopy shows multiple esophageal rings with raised white specks, longitudinal furrows, and friable esophageal mucosa. Histologic examination of the mucosa shows intense inflammation of the lamina propria with more than 15 eosinophils per high-power field. No strictures are detected.

Which of the following is the most appropriate management for this patient?
AEndoscopic esophageal dilatation
BLeukotriene receptor antagonist therapy
COral topical (swallowed) corticosteroid therapy
DProton pump inhibitor therapy
Correct Answer: C)

Educational Objective:Treat eosinophilic esophagitis.
Key Point

* Swallowed liquid corticosteroids are the first-line therapy for eosinophilic esophagitis.

Eosinophilic esophagitis is characterized by eosinophilic infiltration of the esophageal mucosa. Clinically, the disease most commonly presents with dysphagia or food impaction in atopic men in their third to fourth decades of life. For many years considered a pediatric disorder, the disease (or at least its recognition) appears to be rapidly increasing in adults. Diagnosis is suggested from clinical context, supported by endoscopic findings of mucosal furrowing or raised white specks (thought to represent eosinophilic microabscesses), and confirmed by histologic examination of the esophageal mucosa.

Besides disimpaction of a food bolus, initial treatment is medical because of concerns about mucosal friability and esophageal perforation in the setting of dilatation. Swallowed topical corticosteroids (fluticasone propionate or beclomethasone) have been shown to produce clinical remission in up to 50% of adults. Proton pump inhibitors do not appear to be effective. Case reports citing the efficacy of leukotriene receptor antagonists have yet to be confirmed in randomized trials.
A 64-year-old man is evaluated for a 3-month history of abdominal bloating and mid-epigastric discomfort associated with a 6.8-kg (15-lb) weight loss. The patient has no significant medical history and takes no medications.

On physical examination, vital signs are normal, and the only significant finding is mild epigastric tenderness.

Laboratory studies:
Complete blood count

Normal
Aspartate aminotransferase

55 U/L
Alanine aminotransferase

67 U/L
Amylase

184 U/L
Lipase

382 U/L

Helical CT scan of the abdomen shows a 2.8-cm pancreatic body mass. There are no liver lesions and no invasion into surrounding major vessels. Endoscopic ultrasonography confirms the presence of an approximately 3-cm lesion without vascular invasion. Fine-needle aspiration specimen is positive for adenocarcinoma.

Which of the following is the most appropriate next step in the management of this patient?
ACombined radiation therapy and chemotherapy
BDistal pancreatectomy
CPalliative care consultation
DPancreatic enzyme supplementation
Correct Answer: B)

Educational Objective:Manage localized adenocarcinoma of the pancreas.
Key Point

* Surgery is the only treatment that provides a potential cure in patients with localized pancreatic cancer, with a 5-year survival rate of 10% to 30%.

At the time of diagnosis, about 80% to 85% of pancreatic cancers are unresectable because of distant metastases or invasion or encasement of the major blood vessels. Treatment of pancreatic cancer that has not metastasized nor spread to the local vasculature is surgical resection, with distal pancreatectomy being the preferred procedure for lesions of the pancreatic body. Evaluation of whether the tumor is resectable pre-operatively is performed with a combination of helical CT of the abdomen and endoscopic ultrasonography. Even with surgery and complete resection of the tumor, the 5-year survival rate is only 10% to 30%.

Concurrent radiation therapy and chemotherapy alone delays disease progression and may improve survival in patients with localized unresectable pancreatic cancer but will not provide a cure in patients with localized resectable pancreatic cancer. Pancreatic enzymes are used in patients with chronic pancreatitis or after pancreatic surgery to treat pancreatic malabsorption. In this otherwise healthy patient with a localized lesion and a potential for curative resection, palliative care is not indicated.
A 37-year-old woman is evaluated for diffuse musculoskeletal pain at her 1-year follow-up after a Roux-en-Y gastric bypass for medically complicated obesity. Before surgery she had type 2 diabetes mellitus for which she required insulin therapy; after surgery the diabetes was manageable with diet modification alone. She also had hyperlipidemia, for which she took a statin, but after surgery her hyperlipidemia resolved and she discontinued statin therapy 6 months ago. She has lost 36.4 kg (80 lb) since surgery but she thinks that her weight has stabilized over the past few months. She has two to three bowel movements of well-formed stool a day. She has some diffuse musculoskeletal pain but otherwise feels well. Her medical history includes the gastric bypass, a cholecystectomy, diet-controlled type 2 diabetes mellitus, and hypothyroidism. Her medications include levothyroxine, a multivitamin containing iron, vitamin B12 by injection, and over-the-counter calcium and vitamin D.

On physical examination, vital signs are normal; the BMI is 29. There is redundant skin over the arms and trunk and mild thyromegaly. There is no jaundice or scleral icterus. Abdominal examination reveals no tenderness or hepatomegaly; there is normal muscle strength with no focal muscle tenderness. Laboratory studies show a serum alkaline phosphatase of 314 U/L; all other tests, including complete blood count, aminotransferases, bilirubin, γ-glutamyltranspeptidase, serum thyroid-stimulating hormone, free thyroxine, and calcium, are normal.

Which of the following is the most appropriate next step in the evaluation of this patient?
AAnti–smooth muscle antibody test
BMeasurement of 25-hydroxyvitamin D
CMeasurement of serum creatine kinase
DMeasurement of triiodothyronine
EUltrasonography of the liver
Correct Answer: B)

Educational Objective:Diagnose vitamin D deficiency after bariatric surgery.
Key Point

* Nutritional deficiencies are common after bariatric surgery, and patients typically need ongoing supplementation with iron, vitamin B12, calcium, and vitamin D.

Patients who have bariatric surgery are at risk for various vitamin deficiencies. After such surgery, most patients require ongoing therapy with a multivitamin containing iron, along with vitamin B12, calcium, and vitamin D. Other requirements include folic acid and vitamin A, which are often adequately replaced through the use of a multivitamin, although additional supplementation may be required. In addition, thiamine should be given for the first 6 postoperative months. This patient is taking calcium and vitamin D supplementation, but over-the-counter vitamin D supplementation may not be of sufficient quantity to sustain the requirements in this patient. Her diffuse musculoskeletal discomfort and elevated serum alkaline phosphatase are likely the result of vitamin D deficiency, with resultant osteomalacia, and, therefore, serum 25-hydroxyvitamin D should be measured.

Anti–smooth muscle antibody is a serologic marker of autoimmune hepatitis; although this disorder may produce abnormal liver chemistry studies, it usually is considered a hepatitic condition with predominant elevation of the aminotransferases rather than cholestasis. Serum creatine kinase may be a reasonable test in a patient who has myalgias while taking a statin, but this patient’s musculoskeletal pain began months after she stopped taking the drug. Although thyroid dysregulation can cause both myalgias and abnormal liver chemistry results, the patient’s thyroid-stimulating hormone and thyroxine levels are within normal limits while she is taking thyroid hormone replacement; therefore, further thyroid function testing is unnecessary. Ultrasonography of the liver would be the next best step in a patient who has serum chemistry results indicative of cholestasis, but with alkaline phosphatase being the only test elevated and without clinical features of biliary disease, nonhepatic causes of an elevated alkaline phosphatase need to be considered.
A 37-year-old man is evaluated to determine his risk for colon cancer because his 62-year-old mother was recently diagnosed with colon cancer. The patient is healthy and takes no medications. He has a bowel movement daily, has not noticed any change in bowel habits, and has never seen blood in his stool. He does not have any abdominal pain, weight loss, or jaundice.

His mother’s cancer was in the ascending colon, and she is recovering from surgery. His maternal uncle was diagnosed with colon cancer at age 48 years, and he died of the disease a year later. His mother’s other two siblings, aged 44 and 46 years, are healthy. His maternal grandfather was diagnosed with colon cancer in his 60s but died of complications of heart disease. His maternal grandmother survived endometrial cancer diagnosed in her 50s and died in her 70s of a stroke. There is no history of cancer on his father’s side.

Which of the following is the most appropriate management for this patient?
ABlood test of the mother to evaluate for microsatellite instability
BColonoscopy when he reaches age 50 years
CFlexible sigmoidoscopy now
DReferral for genetic counseling
Correct Answer: D)

Educational Objective:Evaluate a patient with a potential familial colorectal cancer syndrome.
Key Point

* Initial genetic testing for colon cancer syndromes is most effective in a patient with cancer rather than in an unaffected family member.

This patient has a family history that meets the Amsterdam criteria for hereditary nonpolyposis colorectal cancer (HNPCC). He should be referred to a genetic counselor to review the family history and be able to address the many issues that surround genetic testing and the implications for both the patient and his family members.

The best approach to test for HNPCC in this case and confirm the diagnosis would be to obtain tumor tissue from the patient’s mother and test it for microsatellite instability. If the test is positive, a blood test can be done in the patient’s mother in an attempt to find a germline mutation in the mismatch repair genes, and if identified, the patient may be tested to determine whether he is a carrier of that mutation. A negative blood test in the mother does not exclude the diagnosis of HNPCC. Screening for colon cancer in HNPCC should be initiated between age 20 and 25 years and requires a complete colonoscopy because cancers frequently occur in the ascending colon.

Genetic testing for colon cancer syndromes should be performed under the guidance of a genetic counselor. The counselor is skilled at appropriate data collection, pedigree analysis, and pre- and posttest counseling. Flexible sigmoidoscopy may identify polyposis for patients with familial adenomatous polyposis; however, it is inadequate screening for other hereditary colorectal cancer syndromes, such as attenuated familial adenomatous polyposis and HNPCC.
A 32-year-old woman is evaluated for a 5.2-cm mass in the right lobe of the liver detected during evaluation for possible appendicitis, which was ruled out. The patient has a history of migraine and a 6-year history of irregular menses, and her medications include sumatriptan as needed and an oral contraceptive pill.

On physical examination, the patient is afebrile; the blood pressure is 110/65 mm Hg, the pulse rate is 80/min, and the respiration rate is 14/min. The BMI is 21. There are no palpable abdominal masses; bowel sounds are normal, and Murphy sign is not present. Laboratory tests are all normal. CT scan of the abdomen shows a well-circumscribed lesion in the right lobe of the liver with an enhancing central scar; the rest of the liver appears to be normal, and there is no intra- or extrahepatic duct dilatation.

Which of the following is the most appropriate management for this patient?
ADiscontinuation of oral contraceptive pills
BEvaluation for liver transplantation
CObservation
DReferral for surgical resection
Correct Answer: C)

Educational Objective:Recognize and manage focal nodular hyperplasia.
Key Point

* Focal nodular hyperplasia is the most common nonmalignant hepatic tumor not of vascular origin and is usually stable and uncomplicated requiring no immediate management.

This young woman has focal nodular hyperplasia, the most common nonmalignant hepatic tumor that is not of vascular origin. This tumor occurs more commonly in women and usually causes no symptoms unless it is very large, in which case an affected patient may experience discomfort from mass effect on nearby organs. The tumors are caused by aberrant blood supply to an area of the liver and classically are differentiated by the appearance of a central scar.

Unlike adenomas, focal nodular neoplastic tumors are not estrogen-sensitive, and therefore, the patient does not need to discontinue oral birth control. In addition, these tumors do not have a malignant potential and do not need to be resected unless they are large and exert mass effect on other organs or cause focal ductal dilatation. This patient, therefore, does not need to be referred for resection. Because this lesion is radiographically consistent with focal nodular hyperplasia rather than hepatocellular cancer, the patient does not have to be referred for evaluation for liver transplantation. Focal nodular hyperplasia is usually stable and uncomplicated, and therefore, no immediate management is required for this patient.
A 25-year-old woman is evaluated for a 2-year history of almost daily bloating and lower abdominal cramping; the symptoms are associated with constipation, relieved with bowel movements, and seem worse when she is under stress. She has one or two small bowel movements a week and often has a feeling of incomplete evacuation. She never has diarrhea and has not had blood in the stool, nocturnal awakening with pain or for bowel movements, or weight loss. She has taken a fiber supplement without relief. The patient is otherwise healthy, and her only medication is an oral contraceptive pill that she has been taking for 1 year. Her mother had a similar condition when she was younger, but both her parents are alive and well.

On physical examination, vital signs are normal; there is mild lower abdominal tenderness with no rebound, guarding, or palpable abdominal masses. Laboratory studies reveal hemoglobin 13.1 g/dL (131 g/L); serum biochemistry tests, including thyroid-stimulating hormone, are normal.

Which of the following is the most appropriate next step in the management of this patient?
AColonoscopy
BCT scan of the abdomen and pelvis
CDiscontinue the oral contraceptive
DReassurance and polyethylene glycol
Correct Answer: D)

Educational Objective:Manage constipation-predominant irritable bowel syndrome.
Key Point

* Irritable bowel syndrome is a clinical diagnosis that can be made confidently when patients meet the Rome III criteria and do not have alarm indicators.

This patient has irritable bowel syndrome. As a young woman, she fits the demographic profile, and she also meets the Rome III criteria, with abdominal pain relieved by defecation and a change in bowel habits. The most recent formal criteria are the Rome II criteria, which require the presence of at least two of three symptoms occurring for 3 months (not necessarily consecutive) during a 12-month period. These symptoms include pain relieved with defecation, onset associated with change in stool frequency, or onset associated with change in the consistency of the stool. In clinical practice, these criteria have a positive predictive value of 98%. Importantly, she has no alarm indicators, including older age, male sex, nocturnal awakening, rectal bleeding, weight loss, or family history of colon cancer. In the absence of alarm symptoms, additional tests have a diagnostic yield of 2% or less. Furthermore, laboratory studies indicate no anemia or thyroid deficiency. Irritable bowel syndrome is a clinical diagnosis, and there are no laboratory, radiographic, or endoscopic findings that aid in diagnosis. Additional evaluation is not only unnecessary and expensive but also potentially harmful, especially when invasive procedures are ordered; additionally, confidence in the diagnosis is undermined when serial testing is ordered. The patient should be reassured that although this problem is annoying and inconvenient, it is not life-threatening. The patient has constipation-predominant irritable bowel syndrome, and her symptoms will likely be alleviated if she has more frequent and satisfying bowel movements. Because fiber supplementation has not been helpful, a nonabsorbed osmotic laxative such as polyethylene glycol will likely provide her significant relief.

There is no indication for the patient to undergo a CT scan or colonoscopy. Oral contraceptives are not typically associated with the syndrome, and she began taking the medication after the onset of her symptoms.
A 26-year-old man is evaluated in the emergency department for shortness of breath and fatigue. He has had maroon stools for the past 2 weeks associated with occasional mild, crampy periumbilical pain but no severe pain. He has had no stools for the past 48 hours.

On physical examination, the temperature is 37.1 °C (98.8 °F), the blood pressure is 94/62 mm Hg supine, the pulse rate is 116/min, and the respiration rate is 14/min; the BMI is 19.5. He has pale conjunctivae and regular tachycardia. The chest is clear and the abdomen is normal with no organomegaly. Examination of the stool is positive for occult blood. Laboratory studies reveal hemoglobin 6.8 g/dL (68 g/L) with a mean corpuscular volume of 74 fL and a platelet count of 446,000/µL (446 × 109/L); serum biochemistry tests, prothrombin time, activated partial thromboplastin time, and INR are normal. Following volume resuscitation, esophagogastroduodenoscopy is normal and colonoscopy shows some old blood but no active bleeding. Wireless capsule endoscopy fails to identify a source of bleeding.

Which of the following is the most appropriate next step in the evaluation of this patient?
ASmall-bowel barium contrast study
BCT scan of the chest and abdomen
CMesenteric angiography
DTechnetium-99m (99mTc) pertechnetate (Meckel) scan
Correct Answer: D)

Educational Objective:Evaluate a young patient with obscure gastrointestinal bleeding.
Key Point

* Bleeding from a Meckel diverticulum may be a source of obscure bleeding in young patients.

In a young patient, the next best test of the options provided to identify a source of bleeding would be a technetium-99m (99mTc) pertechnetate scan (Meckel scan) to identify the presence of Meckel diverticulum. Meckel diverticulum is the most common congenital anomaly of the gastrointestinal track and is located near the ileocecal valve. It often contains heterotopic gastric mucosa that ulcerates and bleeds. Technetium-99m (99mTc) pertechnetate has an affinity for the gastric mucose, and the Meckel scan identifies the heterotopic mucosa. Bleeding from a Meckel diverticulum is common in children but is an uncommon cause of blood loss in adults.

CT scan does not provide a luminal view of the intestine and is not likely to identify a bleeding source in this patient. Mesenteric angiography is not the appropriate next step because the patient does not have active, ongoing, brisk bleeding that would be detected and treated with angiography. A small-bowel barium study would not be a good choice; it obscures endoscopic visualization, has poor sensitivity for identifying sources of blood loss, and does not offer therapeutic options.
A 58-year-old man is evaluated for a 4-week history of progressive difficulty swallowing liquids. The symptom begins immediately with the initiation of a swallow and has recently been associated with coughing. The patient does not have fever or weight loss. His only significant medical history is atherosclerotic cardiovascular disease; his only medications are a β-blocker, a statin, and aspirin.

Vital signs are normal, and physical examination shows only crackles at the posterior base of the right lung field.

Which of the following is the most appropriate next diagnostic test in the evaluation of this patient?
AAmbulatory esophageal pH monitoring
BCT scan of the chest and abdomen
CEsophageal manometry
DVideofluoroscopy
Correct Answer: D)

Educational Objective:Evaluate dysphagia.
Key Point

* In patients with dysphagia, immediate onset of symptoms with initiation of a swallow, difficulty swallowing liquids, and cough associated with the swallowing difficulty are characteristic of oropharyngeal rather than esophagealdysphagia.

The diagnosis and management of swallowing difficulties depend initially on determining whether the patient’s symptoms are characteristic of either oropharyngeal dysphagia or esophageal dysphagia. The cornerstone of this determination is the medical history. In this patient, the immediate onset of symptoms with the initiation of a swallow, the difficulty swallowing liquids, and the cough associated with the swallowing difficulty make oropharyngeal, or transfer, dysphagia more likely than esophageal dysphagia. Videofluoroscopy, which allows real-time radiographic analysis of swallowing function, is the most sensitive test for oropharyngeal dysphagia. The procedure can show abnormal movement of a bolus, such as aspiration, pooling in pharyngeal recesses, movement of anatomic structures, muscle activities throughout the area, and oral and pharyngeal transit times. Common causes of oropharyngeal dysphagia are stroke, Parkinson disease, amyotrophic lateral sclerosis, myasthenia gravis, and muscular dystrophy.

Ambulatory esophageal pH monitoring is the most accurate technique to diagnose gastroesophageal reflux disease but is not the study of choice for evaluation of oropharyngeal dysphagia. CT scan of chest and abdomen can be an important adjunct to the evaluation of patients with esophageal dysphagia, because extraluminal causes of obstruction can be detected by cross-sectional imaging. Esophageal manometry is used in the diagnostic evaluation of patients with motility disorders of the esophagus. Although patients with such disorders may present with dysphagia of both solids and liquids, the immediate onset of symptoms with swallowing makes oropharyngeal dysphagia more likely.
A 48-year-old man is evaluated 8 weeks after having been hospitalized for an episode of sudden nausea, hematemesis, and profound hypotension. He was treated in the emergency department with fluids and an erythrocyte transfusion. An esophagogastroduodenoscopy showed a 1.5-cm clean-based ulcer in the duodenal bulb; biopsy specimens of the body of the stomach and antrum showed chronic active gastritis and the presence of Helicobacter pylori. He was discharged from the hospital and prescribed a 10-day course of omeprazole, clarithromycin, and amoxicillin. He had taken omeprazole for the past 8 weeks and has just completed therapy before his visit. The patient’s father had gastric cancer.

On examination, the patient is asymptomatic and afebrile; the blood pressure is 118/72 mm Hg and the pulse rate is 88/min. Physical examination and complete blood count are normal.

Which of the following is the most appropriate next step in the management of this patient?
A Helicobacter pylori serum antibody test
B Helicobacter pylori urea breath test in 8 weeks
C Helicobacter pylori urea breath test now
DRepeat esophagogastroduodenoscopy with biopsies
ENo further testing
Correct Answer: B)

Educational Objective:Evaluate for eradication of Helicobacter pylori infection.
Key Point

* The two noninvasive tests for determining Helicobacter pylori eradication are the stool antigen test and the urea breath test; the sensitivity of breath testing is adversely affected by recent therapy with a proton pump inhibitor or antibiotic.

Although testing for eradication of Helicobacter pylori is not cost-effective or practical in most cases, in this patient with a history of significant gastrointestinal bleeding from H. pylori infection requiring hospitalization and transfusion, eradication testing would be important to prevent future complications. In addition to patients with H. pylori-associated ulcers, eradication testing should be considered in patients with previous early-stage gastric cancer, H. pylori-related MALT lymphoma, or persistent dyspepsia after a test-and-treat approach. The father’s history of gastric cancer would be another compelling reason to evaluate for eradication of the organism in this patient. Urea breath testing is a noninvasive test for evaluation of H. pylori eradication, but the sensitivity of the test can be markedly reduced in patients taking a proton pump inhibitor or recent antibiotics, and therefore, waiting to perform this test in 4 to 8 weeks after recent discontinuation of omeprazole would be recommended rather than testing now, which may lead to false negative results.

Although serum antibody testing is widely available and has good negative predictive value, it is not recommended after therapy because sustained antibody titers cannot easily distinguish between active and treated infection. Because this patient had a duodenal rather than gastric ulcer, a follow-up esophagogastroduodenoscopy to assess for healing and rule out malignancy is not required; in addition, repeating the procedure to evaluate for H. pylori eradication would not be a cost-effective testing method and would only be recommended if additional histopathologic evaluation were needed, for example, if dysplasia had been found initially. To do no further testing would not be recommended in this patient with a significant H. pylori-associated ulcer bleed.
A 51-year-old man is evaluated for an 8-month history of mid-epigastric pain that is worse after eating, six to eight bowel movements a day usually occurring after a meal, and loss of 6.8 kg (15 lb) over the past 6 months. The patient drinks six to eight cans of beer a day. He takes no medications.

On physical examination, the patient is thin (BMI 21) and has normal bowel sounds, mid-epigastric tenderness, but no evidence of hepatosplenomegaly or masses. Rectal examination reveals brown stool that is occult blood negative. The remainder of the examination is normal. Plain radiograph of the abdomen shows a normal bowel gas pattern and is otherwise normal.

Laboratory studies:
Leukocyte count

6800/µL (6.8 × 109/L)
Platelet count

69,000/µL (69 × 109/L)
Fasting plasma glucose

104 mg/dL (5.77 mmol/L)
Aspartate aminotransferase

191 U/L
Alanine aminotransferase

82 U/L
Amylase

122 U/L
Lipase

289 U/L

Which of the following tests is most likely to establish the diagnosis in this patient?
AColonoscopy
BCT scan of the abdomen
CMeasurement of serum antiendomysial antibodies
DStool for leukocytes, culture, ova, and parasites
Correct Answer: B)

Educational Objective:Diagnose chronic pancreatitis.
Key Point

* Patients with chronic pancreatitis present with abdominal pain and in more severe cases with malabsorption and endocrine insufficiency.

This patient has chronic pancreatitis secondary to alcohol abuse, which has resulted in malabsorption. The three classic findings in chronic pancreatitis are abdominal pain that is usually mid-epigastric, postprandial diarrhea, and diabetes mellitus secondary to pancreatic endocrine insufficiency. Malabsorption occurs in patients with chronic pancreatitis when approximately 80% of the pancreas is destroyed. Malabsorption presents with diarrhea and steatorrhea, weight loss, and deficiencies of fat-soluble vitamins because the damaged pancreatic gland is no longer producing the pancreatic exocrine enzymes to absorb food. Patients with a typical presentation may not need additional testing. However, most patients with chronic pancreatitis have only nonspecific abdominal pain and require diagnostic radiographic imaging studies. The presence of pancreatic calcifications on radiographs confirms the diagnosis. Plain films of the abdomen will show pancreatic calcifications in approximately 30% of patients. Most patients, however, require abdominal CT scans, which are able to detect pancreatic calcification in up to 90% of patients. CT scanning can also exclude other causes of pain. Radiographic evidence of pancreatic ductal dilation, pseudocysts, or mass lesions may also help identify the cause of pain and determine the type of therapy.

Antiendomysial antibodies are a marker for celiac disease, which is unlikely in this patient with an evident history of pancreatic malabsorption. Although colonoscopy is indicated as a screening tool for average risk asymptomatic patients beginning at the age of 50 years and for patients with a change in bowel habits and weight loss, this patient’s history suggests pancreatic malabsorption and colonoscopy is less likely than abdominal CT scan to confirm the diagnosis. Stool studies are appropriate for determining the cause of an acute infectious diarrhea, but this patient has had diarrhea for 8 months and infectious diarrhea is not usually associated with such a degree of weight loss.
A 32-year-old man is evaluated for a 2-week history of nausea, malaise, low-grade fever, vomiting, and jaundice. He has no other significant medical history and takes only ibuprofen for headache and fever.

On physical examination, the temperature is 37.6 °C (99.7 °F), the blood pressure is 110/75 mm Hg, the pulse rate is 90/min, and the respiration rate is 22/min; the BMI is 25. Examination reveals scleral icterus, jaundice, 1+ pitting lower extremity edema, hepatomegaly, mild asterixis, and somnolence. There are no stigmata of chronic liver disease.

Laboratory studies:
Bilirubin (total)

17.5 mg/dL (393.3 µmol/L)
Bilirubin (direct)

7.2 mg/dL (123.1 µmol/L)
Aspartate aminotransferase

8790 U/L
Alanine aminotransferase

7650 U/L
Alkaline phosphatase

195 U/L
INR

2.3
Hepatitis B surface antigen

Positive
Hepatitis B core antigen (IgM)

Positive
Hepatitis C antibody

Negative
Hepatitis A total antibody

Positive
Blood alcohol

Negative
Acetaminophen

Undetectable

Ultrasonography shows hepatomegaly and increased echogenicity, a normal spleen, and perihepatic ascites. There is no ductal dilatation.

Which of the following is the most appropriate management for this patient?
ABegin interferon therapy
BEvaluation for liver transplantation
CEndoscopic retrograde cholangiopancreatography
DCorticosteroids
Correct Answer: B)

Educational Objective:Manage fulminant liver failure.
Key Point

* About 5% patients with acute hepatitis B virus infection develop acute progressive hepatitis B with fulminant hepatic decompensation and need urgent liver transplantation.

The markedly elevated aminotransferase levels, positive hepatitis B surface antigen, and IgM antibody to hepatitis B core antigen establish the diagnosis of acute hepatitis B. Patients at greatest risk for exposure to hepatitis B virus (HBV) infection are those with a history of multiple sexual partners and injection drug users. The course of HBV infection depends mainly on the age at which a patient is exposed. Most patients who are first exposed as adults develop flu-like symptoms with malaise, nausea, vomiting, and diarrhea with associated jaundice. Children on the other hand most often do not develop symptoms after exposure to HBV. Despite the presentation of an acute infection, most adult patients will clear their infection after a few months. Therefore, most patients can be monitored as outpatients and treated for symptoms only. However, about 5% patients develop acute progressive hepatitis B with hepatic decompensation and need urgent liver transplantation. These patients tend to have an elevated INR and a rising bilirubin level and may develop encephalopathy, a marker of fulminant hepatic failure. This subgroup of patients needs to be recognized, admitted to the hospital, and started on antiviral therapy. Telbivudine, lamivudine, adefovir, and entecavir are acceptable options. Transplantation evaluation should be pursued. Transplantation should be performed when a poor outcome is anticipated but before the patient develops uncontrolled sepsis or prolonged periods of increased intracranial pressure that would preclude recovery even if a functioning liver is transplanted. In the United States, patients with fulminant hepatic failure are given highest priority for transplantation and often receive a transplanted liver within 1 week of evaluation.

Interferon should be avoided in patients with acute hepatitis B and particularly in fulminant hepatic failure due to hepatitis B because of the increased risk of hepatic necroinflammation. The patient has no evidence of ductal dilatation on ultrasonography, and therefore, there is a low yield and high risk to proceed with endoscopic retrograde cholangiopancreatography. Corticosteroids have been studied as a treatment for fulminant hepatic failure and have been found to be not effective. In addition, corticosteroid use is associated with an increased risk for infectious complications.
A 55-year-old woman is evaluated for elevated liver chemistry tests detected on examination for life insurance. She has no symptoms of liver disease and no history of jaundice, ascites, lower extremity edema, or encephalopathy. She used recreational injection drugs between the ages of 20 and 25 years. She has no significant medical history and takes no medications. She drinks about six cans of beer a day.

On physical examination, vital signs are normal. There are spider angiomata on the upper body and the presence of a nodular liver edge and splenomegaly.

Laboratory studies:
Platelet count

88,000/µL (88 × 109/L)
INR

1.4
Bilirubin (total)

1.1 mg/dL (18.8 µmol/L)
Aspartate aminotransferase

48 U/L
Alanine aminotransferase

96 U/L
Alkaline phosphatase

186 U/L
Albumin

3.6 g/dL (36 g/L)
Hepatitis B surface antigen

Negative
Hepatitis C virus antibody

Positive
HCV RNA in serum

500,000 copies/mL

Ultrasonography shows coarsened hepatic echotexture; CT scan shows changes in the liver consistent with cirrhosis and splenomegaly.

Which of the following is the most appropriate next step in the management for this patient?
AEsophagogastroduodenoscopy
BEvaluation for liver transplantation
CLamivudine
DPegylated interferon and ribavirin
Correct Answer: A)

Educational Objective:Evaluate chronic hepatitis C virus infection.
Key Point

* Variceal hemorrhage occurs in 25% to 40% of patients with cirrhosis; upper endoscopy is indicated in patients with newly diagnosedcirrhosis.

This patient presented with asymptomatic elevation of hepatic biochemical tests; further testing demonstrated the presence of hepatitis C virus infection and signs of cirrhosis (splenomegaly, nodular liver contour, thrombocytopenia). Variceal hemorrhage occurs in 25% to 40% of patients with cirrhosis, and since mortality from variceal bleeding is high, screening for varices is a high priority task in all patients with newly discovered cirrhosis. Esophagogastroduodenoscopy is the next best management option to exclude the presence of esophageal varices. Depending on the size and appearance of the varices and the patient’s Child-Pugh grade, prophylactic treatment may be indicated.

Patients with hepatitis C may be candidates for treatment with pegylated interferon and ribavirin; however, in this patient, cirrhosis and its potential complications must be assessed before therapy is begun. Moreover, treatment efficacy is lower in patients with cirrhosis than in patients without cirrhosis. Liver transplantation is the treatment of choice for suitable candidates with end-stage liver disease, fulminant hepatic failure, and certain metabolic disorders. Although the patient appears to have cirrhosis, there is no ascites, jaundice, or hepatic dysfunction that is severe enough to warrant consideration of transplantation. Treatment is recommended for all patients with chronic hepatitis B who have abnormal serum alanine aminotransferase values, HBV DNA levels >105 copies/mL, and liver biopsy findings of active disease. The oral agents lamivudine, adefovir, and entecavir are safe and well tolerated and may improve liver function in patients with decompensated hepatitis B. However, there is no indication that this patient has hepatitis B and therapy with lamivudine is not indicated.
A 25-year-old man is evaluated after being turned down as a blood donor because of abnormal liver chemistry tests. The patient is healthy, takes no medications, does not smoke, and drinks alcohol socially. His parents and siblings are alive and healthy; his maternal grandfather developed type 2 diabetes mellitus at age 75 years. The review of systems is normal.

On physical examination, vital signs and BMI are normal.

Laboratory studies:
Hemoglobin

11.9 g/dL (119 g/L)
Mean corpuscular volume

76 fL
Cholesterol (total)

155 mg/dL (4.01 mmol/L)
LDL cholesterol

85 mg/dL (2.2 mmol/L)
HDL cholesterol

33 mg/dL (0.85 mmol/L)
Bilirubin (total)

0.5 mg/dL (8.55 µmol/L)
Aspartate aminotransferase

25 U/L
Alanine aminotransferase

58 U/L
Alkaline phosphatase

110 U/L

Serologic tests for hepatitis virus infection are normal.

Which of the following is the most appropriate diagnostic test for this patient?
AAnti-tissue transglutaminase antibody
Bα1-Antitrypsin concentration
CBlood alcohol level
DLiver biopsy
Correct Answer: A)

Educational Objective:Diagnose asymptomatic celiac disease.
Key Point

* Measurement of serum anti-tissue transglutaminase antibodies has a sensitivity and specificity of approximately 90% for celiac disease.

Celiac disease is a small-bowel disorder characterized by mucosal inflammation, villous atrophy, and crypt hyperplasia, which occur on exposure to gluten. The disease is rather common, affecting nearly 1% (1/133 persons) of the population. Although many affected patients have diarrhea and steatorrhea, as well as bloating, abdominal pain, and malabsorption of vitamins and minerals, other patients are asymptomatic at diagnosis. Such patients may be found to have anemia or osteoporosis as part of routine health maintenance testing. Still others present with neurologic symptoms, dermatitis herpetiformis, or elevated concentrations of liver enzymes. The elevated alanine aminotransferase in this patient should normalize with a gluten-free diet if he has celiac disease. The patient’s elevated alkaline phosphatase concentration is likely the result of increased bone turnover as a result of vitamin D and calcium malabsorption. Celiac disease is one of the most common causes of osteoporosis in men. Measurement of serum anti-tissue transglutaminase antibodies has a sensitivity and specificity of approximately 90% for celiac disease.

α1-Antitrypsin deficiency affects approximately 1 in 1600 persons, and although measurement of α1-antitrypsin is an appropriate test in the work-up of abnormal liver function tests, it is not the next best test because of the low prevalence of the disease compared to celiac disease, and it cannot explain the patient’s anemia. The patient admits to social use of alcohol, and this is to be strictly avoided in the setting of liver disease. Unless the patient was actively drinking alcohol around the time of the testing, measuring the blood alcohol would not be helpful. Furthermore, in alcoholic liver disease, the aspartate aminotransferase concentration is usually greater than the alanine aminotransferase. Liver biopsy would be an extreme measure to take as a result of a mildly abnormal set of liver chemistry tests and is not indicated as an initial step.
A 78-year-old woman is evaluated in the hospital after being admitted 5 days ago for a 2-week history of abdominal pain and nausea, along with black, tarry stools for the past 36 hours. On day 1 esophagogastroduodenoscopy showed a clean-based bleeding gastric ulcer that was positive for Helicobacter pylori infection; the ulcer was treated with injection therapy and coagulation therapy with probe cautery, and proton pump inhibitor therapy was initiated. The bleeding did not stop, and esophagogastroduodenoscopy was repeated on day 3 with endoclip therapy. The bleeding continued, and the patient has received eight units of packed erythrocytes.

On physical examination on day 5, the temperature is 37.2 °C (99.0 °F), the blood pressure is 95/50 mm Hg, the pulse rate is 103/min, and the respiration rate is 16/min. Rectal examination reveals melanotic stool. Laboratory studies reveal hemoglobin of 10.8 g/dL (108 g/L); all other tests, including coagulation parameters, are normal.

Which of the following is the most appropriate next step in the management of this patient?
ABleeding scan
B Helicobacter pylori eradication therapy
CIntravenous octreotide
DSurgery
Correct Answer: D)

Educational Objective:Manage acute gastrointestinal bleeding that does not respond to endoscopic therapy.
Key Point

* Surgical treatment of upper gastrointestinal bleeding should be considered when endoscopic therapy fails.

Most cases of upper gastrointestinal bleeding can be effectively treated with medical and endoscopic therapy. The most common need for surgical treatment in such bleeding is failure of the bleeding to respond to endoscopic treatment. Surgical consultation should also be considered in patients who rebleed, in patients who require a large number of transfusions, and for large ulcers of the lesser curve of the stomach or posterior wall of the duodenum.

The diagnosis of bleeding from a gastric ulcer has already been made in this patient, and therefore no further diagnostic modalities such as a bleeding scan are required. Some studies have suggested the use of intravenous octreotide in the use of peptic ulcer bleeding, but no data supports its effectiveness after endoscopic therapy fails. In patients with Helicobacter pylori, eradication prevents ulcer recurrence but has no effect on the outcome of patients with acute upper gastrointestinal bleeding. However, all patients with bleeding peptic ulcers should receive high-dose proton pump inhibitor therapy. A meta-analysis showed that adjuvant high-dose proton pump inhibitor therapy following endoscopic hemostasis for ulcers at high risk of rebleeding reduces rebleeding, surgery, and mortality.
A 45-year-old man is brought to the emergency department by fire rescue after his wife found him in bed lethargic and disoriented. The previous 2 days, the patient had been agitated and irritable and his speech was slurred. The patient has a history of cirrhosis complicated by esophageal variceal bleeding, ascites, and lower extremity edema, and his medications are furosemide, spironolactone, propranolol, and lactulose. He has a long history of alcohol dependence but has been sober for 1 year.

On physical examination, the patient is somnolent but arousable although not responsive to commands; he is afebrile. The blood pressure is 100/78 mm Hg, the pulse rate is 65/min, and the respiration rate is 12/min; the BMI is 26. There are no focal neurologic deficits; the pupils are equal and reactive to light. There is shifting abdominal dullness and 2+ lower extremity edema. The stool is negative for occult blood.

Laboratory studies:
Leukocyte count

5600/µL with normal differential (5.6 × 109/L)
Glucose (random)

112 mg/dL (6.21 mmol/L)
Sodium

135 meq/L (135 mmol/L)
Potassium

3.5 meq/L (3.5 mmol/L)
Chloride

100 meq/L (100 mmol/L)
Bicarbonate

28 meq/L (28 mmol/L)
Bilirubin (total)

4.0 mg/dL (68.4 µmol/L)
Bilirubin (direct)

2.3 mg/dL (39.3 µmol/L)
Aspartate aminotransferase

78 U/L
Alanine aminotransferase

45 U/L
Alkaline phosphatase

167 U/L
Albumin

2.7 g/dL (27 g/L)
Ammonia

230 µg/dL (135 µmol/L)
Blood alcohol

Negative

Urinalysis is negative. Dipstick is positive for 3+ leukocyte esterase and nitrites. CT scan of the head is normal. A diagnostic peritoneal fluid tap excludes spontaneous bacterial peritonitis. Empiric antibiotic therapy is started.

Which of the following is the most appropriate management for this patient?
ACorticosteroids
BIncrease lactulose therapy
CIntravenous albumin
DTransjugular intrahepatic portosystemic shunt (TIPS)
Correct Answer: B)

Educational Objective:Manage hepatic encephalopathy.
Key Point

* First-line therapy for hepatic encephalopathy is lactulose.

This patient has severe encephalopathy manifested by worsening somnolence. The patient began manifesting signs of increasing encephalopathy days prior to the admission, when his wife noticed that he was becoming more agitated and slurring his speech. Encephalopathy progresses from subtle findings, such as reversal of the sleep-wake cycle or mild mental status changes, to irritability, confusion, slurred speech, and ultimately coma if not recognized and treated. There can be multiple inciting causes of encephalopathy in patients with cirrhosis, including dehydration, infection (especially spontaneous bacterial peritonitis), diet indiscretions, gastrointestinal bleeding, and medications. This patient likely became worse with the development of the urinary tract infection.

The best course of management is to treat the infection and to discontinue the diuretics and increase the lactulose to respond to the encephalopathy. The dose of lactulose should be titrated to achieve two to three soft stools per day with a pH below 6.0. Approximately 70% to 80% of patients with hepatic encephalopathy improve on lactulose therapy, and treatment is usually well tolerated.

Corticosteroids have no role in the reversal of hepatic encephalopathy. Although albumin therapy is often instituted in patients hospitalized with infections, especially spontaneous bacterial peritonitis and associated dehydration, it is not a primary treatment of encephalopathy. Transjugular intrahepatic portosystemic shunt (TIPS) is also not appropriate because placement of TIPS is likely to precipitate worsening hepatic encephalopathy as more blood is bypassed through the shunt rather than processed by the liver.